初二英语寒假作业(一)

初二英语寒假作业(一)

一. 词汇.

A.根据所给中文写出正确的单词。

1. We got off the ____________________(长途汽车)and went into the park.

2.What do you think is the most ___________________(不重要的) in poor countries ?

3.I hope you can give me some ______________________(建议).

4.When he is hungry ,he always_____________________(敲)on the cage.

5.They are all the ___________________(英雄) in the history.

7. We must remember those ____________________.

8. You can have some ____________________lessons at the school.

9. He doesn‟t know how to look after____________________ .

10. Do you think Betty is __________________ of her three sisters?

12._________________________in a big city is very interesting.

13. I________________________ an interesting film at present.

14. He never __________________ out at night.

15. David________________________ off the bus yesterday.

二. 选择题

( ) 1. ---Mum, may I watch TV now?

---No, you finish your homework first.

A. mustn‟t B .must C. need D. can‟t

( ) 2. ---You look today, Julia!

---Yeah, I‟ve got an MP3.It‟s a birthday present from my parents.

A. sad B. worried C. happy D. tried

( ) 3. ---Why did you come to school late this morning?

I watched the Football World Cup until 12 o‟clock last night.

A. If B. Because C. Since D. though

( ) 4. It is ____useful book. 1 borrowed it from ____school library. A. an; a B. a; an C. a; the D. an; the

( ) 5. We can do the work better with____ money and ____ people.

A. less, fewer B. fewer, less C. less, less D. fewer, fewer

( ) 6. While I____ with my friend, she came in.

A. am talking B. was talking C. talked D. am going to talk

( ) 7. All of us enjoy playing computer games, ___ we can't spend too much time on it.

A. and B. or C. but D. so

( ) 8. Would you please tell me____?

A. what was her name B. what her name was

C. what is her name D. what her name is

( ) 9.— We can use QQ to talk with each other on the Internet.

— Really? Will you please show me ____ it?

A. what to use B. how to use C. how can I use D. what can I use

( ) 10.—Mum, could I have an MP3 like this?

—Certainly, we can buy one, but as good as this. The price of this kind is a little high.

A. a cheap B. a cheaper C. a small D. a smaller

( ) 11.—Can I help you? —I‟d like A. two pair of shoes B. two pairs of shoe

C. two pair of shoe D. two pairs of shoes

( ) 12.—You watched the fashion show last night, didn‟t you?

— I missed the beginning.

A. so B. and C. but D. then

( ) 13.I feel it hard to keep up with my classmates. But whenever I want to , my teacher always encourages me to work

harder.

A. go on B. give up C. run away D. give back

( ) 14.How time flies! Three years A. is B. are C. was D. were

( ) 15.---Mum, we‟re going to visit an island this weekend. --- !

A. Goodbye B. That‟s OK C. Not at all D. Have a good trip

三. 完型填空

You really have to get very old before you find you're old. I'm in my middle fifties and I don't feel old yet. However, sometimes I look

2. A. today's

3. A. gone

4. A. made

5. A. making

6. A. untrue

7. A. movie

8. A. lost

9. A. As usual

10. A. movies

四. 阅读理解 B. yesterday's B. new B. decided B. watching B. loud B. television B. increased B. Above all B. food C. tomorrow's C. expensive C. hoped C. preparing C. high C. radio C. watched C. Besides C. cars D. future's D. popular D. tried D. buying D. realistic D. telephone D. played D. Especially D. televisions

A

When Kyle walked into Ernie‟s Pet World, he looked very anxious. The shop owner, Ernie, jumped out of his seat to greet Kyle. He was the shop‟s first customer of the day.

“Good morning, sir!” Ernie said, “What can I help you with?”

“Well, I ……”Kyle started to say.

“Wait, don‟t tell me,” Ernie stopped him. “You‟re looking for a …a little dog… for your daughter‟s birthday, Right?”

“Not really, I just….”

Ernie didn‟t let him finish. “Ah, I‟ve got it, You just moved to a new office, and you want some fish for it, I have some very nice fish bowls over here.”

“In fact, I…”Kyle was starting to look very nervous and strange.

“No fish? Ah, a cat! You look like a cat person. At Ernie‟s Pet World, we have the best cats. Take a look at this lovely one with long, white hair, She‟s looking at you. She‟s thinking, „Take me home. Take me home.‟ Would you like some cat food and toys, too?”

“No, thank you,” Kyle said. By this time, he was walking up and down. He had a terrible look on his face. “Really, I‟m not interested in cats or fish or little dogs.”

“What do you want, then?” Ernie asked.

Kyle looked like he was going to cry. “I just want to know if can use your toilet!” he said at last.

( )1. Which is TRUE about Ernie‟s Pet World?

A. It has many workers. B. Kyle goes there often.

C. It sells school things. D. Ernie is the owner of the shop.

( )2. What kind of person is Ernie?

A. Very lazy. B. Rather shy. C. Over warm-hearted. D. Not friendly.

( )3. Kyle went into the Ernie‟s Pet World to .

A. buy a little dog B. take fish home

C. meet the shop owner D. look for a washroom

( )4. What does the underlined word “anxious” mean in Chinese?

A.焦急的 B.兴奋的 C.生气的 D.失望的

( )5. What is the best title of the story?

A. Kyle and Ernie B. Pet or Toilet

C.A Pet Lover D.A Busy Pet Shop

B

Do you know Fei Junlong and Nie Haisheng? Fei Junlong was born in 1965. He is a Chinese astronaut selected(选拔)as a member of the Shenzhou program. He was born in Jiangsu and joined the People‟s Liberation Army Air Force(中国人民解放军空军)in 1982 at the age of 17. And since then he has only returned home six times. As a fighter pilot(战机飞行员), he was selected to an astronaut in 1998.

He married Wang Jie in 1991 and has one son.

Nie Haisheng was born in September 1964. He is also a Chinese astronaut selected for the Shenzhou program. He is from Hubei. After graduating from high school he joined the People‟s Liberation Army Air Force, and became a fighter pilot. In 1998 he was selected for the astronaut program.

He was one of the three astronauts who were in the final group to train for the Shenzhou 5 fight. Yang Liwei was also picked out for the flight, with Zhai Ahigang.

On September 23,2005 it was reported that Zhai and Nie would be one of the three pairs of astronauts who would be in the final training for Shenzhou 6. But in the end, Fei and Nie were chosen, and Zhai missed his chance to space again.

Nie Haisheng got married to Nie Jielin and has an 11-year-old daughter.

( ) 6. When did Nie join the army?

A. In 1982. B. In 1998.

C. In 1991. D. After his graduation from high school.

( ) 7. Who among the astronauts in this passage hasn‟t been to space?

A.Zhai Zhigang. B.Nie Haisheng. C.Yang Liwei. D.Fei Junlong.

( ) 8. How many times has Fei Junlong returned home since he left at the age of 17?

A. Three times. B. Four times. C. Six times. D. Eight times.

( ) 9.How many people did the writer mention in this passage?

A.5. B.6. C.7. D.8.

( ) 10.Which of the following is TRUE according to this passage?

A. Yang Liwei was born in Jiangsu.

B. Nie Haisheng has an elever-year-old daughter.

C. Fei Junlong is one year older than Nie Haisheng.

D. Zhai Zhigang got married to Nie Jielin and has a son.

阅读下面短文,根据短文内容回答下列各题。

11.What telephone number should you call if you need a 2-bedroom flat?

_______________________________________________________________________________

12.Mr. Chen lives in Hong Kong. He has never traveled abroad before and he wants to make a tour around Europe. Which travel company can help him?

_______________________________________________________________________________

13.How much does it cost to make a seven-day tour around five countries in Europe?

_______________________________________________________________________________

14.If Mr. Bell wants to be a business journalist for World watch, what languages does he have to know well?

_______________________________________________________________________________

15.If Miss Jones has worked as a journalist for one year, can she be employed(聘用)by World watch?

_______________________________________________________________________________

五. 作文

请以 “My life school”为题写一篇80词左右的文章。

提示词:get up , have breakfast , go to school , have lessons ,

practice…doing , do…activity ,do…exercise ,

chat…with ,listen/watch , enjoy ,

初二英语寒假作业(二)

一、词形变化 根据句意﹑汉语注释及所给单词,在空格内写出各单词的正确形式。

1. Tim and I are good friends, and we keep our secrets to _________ (we).

2. The girl lost her money on her way home yesterday. How _______ she was! (care)?

3. My cat weighs two kilos. But I don‟t know my son‟s ___________. (weigh)

4. Tom has the _________ milk of us three. (little)

5. Amy is becoming much _________after playing softball. (health)

6. ________ (幸运的是), he was not hurt badly.

7. They got the first place, so they were the __________(获胜者)of the football final.

8. We will remember the _________ (令人愉快的)trip forever because we really had a good time during the trip.

9. Jim is ill. He has to take some ________( 药 )three times a day.

10. The 广告 )in this newspaper are more interesting than those in that one.

二、单项选择 在A、B、C、D四个选项中,选出可以填入空白处的最佳选项。

( )1. John is ______ honest boy. He has ______useful book.

A. an ; an B .a ;a C .a ;an D .an ;a

( )2. Most of the workers like driving or riding to work,______ I like walking.

A. and

A. Let‟s go B. so C. but D. or B. I‟d like to

D. It‟s a pleasure

B. What; worse ( )3.---Why not join us in the game, Kitty ?---________, but I have to do my homework first. C. Yes, please A. What a ; bad ( )4. --________ terrible weather it is! --The radio says that it‟ll get _________later.

C. How; bad D. How a ; worse

( )5. Is there _______ about animals on the Internet?

A. special something B. something special

C. special anything D. anything special

( )6. Don‟t speak to him like that , he‟s only an___________ boy.

A. 8 years old B. five-year-old

C. eleven-year-old D. eight-years-old

( )7. The number of the students in Woodland School__________ than before.

A. are more B. are larger C. is fewer D. is smaller

( )8. I don‟t want the bananas. They smell _____.

A. well B. good C. terrible D. badly

( )9. Do you think some special dogs can do things __________ man?

A. as well as B. as good as C. as soon as D. as careful as

( )10. What did you do in the garden? I watched John ______ his bike.

A. to mend B. mended C. mend D. mends

( )11. The trip from the school _____ about half an hour by coach. It was too ______.

A. spends; bored B. takes; bored C. spent ; boring D. took; boring

( )12. ----How did Daniel 1earn how to make a homepage? ----Oh,.

A.taught himself B.learnt himself C.learn by himself D.taught by himself

( )13. He looks so tired ______ too much computer work.

A. because his B. because of C. because it‟s D. because for

( )14. ----Which of the two boxes is heavier? ----This one is heavier than A. another B. the other C. others D. other

( )15. He ________ his homework today.

A. need to finish

三、动词填空 用括号内所给动词的适当形式填空。

1. The 2008 Olympic games_____(take) place in Beijing ,all the Chinese are looking forward to it.

2. The Greens will fly back to their home tomorrow, so they are busy ______(pack) their bags.

3. It is important for wild animals __________ (build) more reserves (自然保护区).

4. Keep ______ (work) on the problem, you will solve it soon. B. needs finishing C. needn‟t to finish D. doesn‟t need to finish

5. Last Sunday, Mum made me ________ (do) some washing at home.

6. It‟s 4:30 p.m. The students of Class 4 _______(play) football on the playground.

7. Mother asks him ______ (not forget) to lock the door when he leaves home.

8. There‟re so many wonderful clothes in the shop, so she can‟t decide which one ____________(choose).

9. They wanted to run away but we _______(catch) them in the end.

10. Linda‟s mother will come back if she ________ (finish) her work this week.

四、完成句子 按所给的汉语,用英语完成下列句子,1-4题每个空格只能填一个单词。

1. 我们应该鼓励孩子们保护动物。

We should__________ children __________ __________ animals.

2. 我愿意和我的朋友分享快乐。

I‟m ______ _______ ________my joy ________ my friend.

3. 我决定呆在家里上网,不出去购物了。

I _______to stay at home and search the Internet _______ _____ _______ shopping.

4. 在下了车之后,他们不再感到恶心了。

After they _______ ________ the coach, they didn‟t feel ______ _______ ________.

5. 西蒙的衬衫和迪克的衬衫颜色一样。

Simon‟s shirt ________________________________________.

6. 你到那儿后,请尽快给我打电话。

After you arrive there, please call me____________________________.

五、完形填空

A teacher was asking a student a lot of questions,.The teacher then decided to ask him some .

“ she asked.

The student for some time and then answered, “An important Person.”

“No.it is a battle(战役).”the teacher said.,it.Then she asked,“Who was the first president of the United States?” The student thought and thought ,but didn‟t.The teacher got very angry and shouted,“George Washington!”

“Come back!” the teacher said.“didn‟t tell you to go.” “Oh,I‟m sorry,” the student said , “I thought you the next student.”

( )1. A.wouldn‟t B.couldn‟t C.shouldn‟t D.mustn‟t

( )2. A.1ittle B.few C.a little D.a few

( )3. A.What B.Which C.Who D.Where

( )4. A.thought B.spent C.waited D.stopped

( )5. A.angry B.worried C.sad D.happy

( )6. A.find B.1ose C.do D.show

( )7. A.speak B.tell C.say D.talk

( )8. A.friend B.seat C.address D.side

( )9. A.He B.She C.I D.We

( )10. A.called B.taught C.needed D.sent

六、阅读理解 阅读下面的短文,然后根据短文内容选择最佳答案。

( A )

A man made a nice talking machine .It could weigh people‟s weight .The man wanted to try the machine before he could make a lot of machines.

He put the machine into the waiting room of a station. There were always lots of people in and out.

The first one who used the machine was an Indian woman. She stood on the machine, the machine thought for a few seconds to decide

5. Last Sunday, Mum made me ________ (do) some washing at home.

6. It‟s 4:30 p.m. The students of Class 4 _______(play) football on the playground.

7. Mother asks him ______ (not forget) to lock the door when he leaves home.

8. There‟re so many wonderful clothes in the shop, so she can‟t decide which one ____________(choose).

9. They wanted to run away but we _______(catch) them in the end.

10. Linda‟s mother will come back if she ________ (finish) her work this week.

四、完成句子 按所给的汉语,用英语完成下列句子,1-4题每个空格只能填一个单词。

1. 我们应该鼓励孩子们保护动物。

We should__________ children __________ __________ animals.

2. 我愿意和我的朋友分享快乐。

I‟m ______ _______ ________my joy ________ my friend.

3. 我决定呆在家里上网,不出去购物了。

I _______to stay at home and search the Internet _______ _____ _______ shopping.

4. 在下了车之后,他们不再感到恶心了。

After they _______ ________ the coach, they didn‟t feel ______ _______ ________.

5. 西蒙的衬衫和迪克的衬衫颜色一样。

Simon‟s shirt ________________________________________.

6. 你到那儿后,请尽快给我打电话。

After you arrive there, please call me____________________________.

五、完形填空

A teacher was asking a student a lot of questions,.The teacher then decided to ask him some .

“ she asked.

The student for some time and then answered, “An important Person.”

“No.it is a battle(战役).”the teacher said.,it.Then she asked,“Who was the first president of the United States?” The student thought and thought ,but didn‟t.The teacher got very angry and shouted,“George Washington!”

“Come back!” the teacher said.“didn‟t tell you to go.” “Oh,I‟m sorry,” the student said , “I thought you the next student.”

( )1. A.wouldn‟t B.couldn‟t C.shouldn‟t D.mustn‟t

( )2. A.1ittle B.few C.a little D.a few

( )3. A.What B.Which C.Who D.Where

( )4. A.thought B.spent C.waited D.stopped

( )5. A.angry B.worried C.sad D.happy

( )6. A.find B.1ose C.do D.show

( )7. A.speak B.tell C.say D.talk

( )8. A.friend B.seat C.address D.side

( )9. A.He B.She C.I D.We

( )10. A.called B.taught C.needed D.sent

六、阅读理解 阅读下面的短文,然后根据短文内容选择最佳答案。

( A )

A man made a nice talking machine .It could weigh people‟s weight .The man wanted to try the machine before he could make a lot of machines.

He put the machine into the waiting room of a station. There were always lots of people in and out.

The first one who used the machine was an Indian woman. She stood on the machine, the machine thought for a few seconds to decide

which language to speak.

“Good morning, Madam.” It said in Indian. “Your weight is 72 kilograms. That‟s three kilograms more .If you eat more fruit and vegetables, you will be soon all right. Please have a nice day.”

The second one to use the machine was a Chinese girl .She stood on the machine and waited to hear her weight.

“Good morning, Miss.” The machine said in Chinese. “Your weight is 45 kilograms. It‟s all right for your age. Keep eating what you eat every day. Please have a nice day.”

The third one to use the machine was a very fat American woman .She thought for a long time to stand on the machine .The machine spoke quickly in English, “Good morning. Will one of you get off?”

1. The machine in this passage could .

A. speak all kinds of languages

C. weigh and talk to people

A. in the station B. tell people what they should eat D. make people laugh . B. in a train

D. in the women‟s room

B. she was a little heavier

D. to eat what she wanted to eat

B. should eat less

D. kept herself healthy

B. She liked thinking.

D. The machine didn‟t like her.

( B )

There are many kinds of animals that live in the forest, such as mice, rabbits, squirrels, bears .

Animals in the forest depend on(依靠)one another for food. Many small forest animals, such as mice, eat green plants for food, and then larger animals eat them. This is called food chain. A food chain begins with the sun. The green plants in the forest use the sun‟s energy to make food for themselves. It‟s called photosynthesis(光合作用). Mice eat flowers and seeds that forest plants produce(生产).The mice get their energy from the plants, and the mice become food for a larger animal, such as the hunting hawk. So the energy from the sun passes from the plants to the mice to the hawk.. A break(中断)in any part of this food chain means some animals may not have enough food to eat.

( )6. Photosynthesis takes place when _____________.

A. larger animals eat small animals.

B. green plants use the sun‟s energy to make food.

C. Forest animals eat the food made by green plants.

D. The food chain is broken.

( )7. What‟s the first link(环) in a food chain?

A. The sun B. The earth C. Green plants D. Forests animals

( )8. Animals sometimes cannot find enough food because _____________.

A. it‟s cloudy.

B. there is very little food in a forest.

C. Photosynthesis stops.

D. There is a break in the food chain.

( )9. What does the food chain show about living things?

A. They depend on each other.

B. Large animals eat small animals.

C. Plants with flowers are the most important.

D. Plants are more than animals.

( )10. What may be the best title of the passage? C. in the doctor‟s waiting room A. she was a little lighter C. it couldn‟t speak Indian A. should eat more 3. The machine told the Indian woman 4. The machine said to a Chinese girl that she C. had to eat more fruit A. She was healthy. C. She was too heavy. 5. How was the American woman?

A. Animals in the world

B. Food chain

C. Plants and man.

D. Animals are in danger.

七、缺词填空 :先通读下面的短文,掌握其大意,然后根据短文内容和所给首字母,在空格内填入一个适当的词,使短文意思完整。

in Russia, he ran short of (缺少)money. So he w $500. “Send the money by telegram(电报)to the bank here. ”After a week Ed began visiting thhis passport (护照)to the bank clerk(职员). “Here is n for you, ”He told him. This went o . He then phoned his brother, asking w the money was. His brother said he had sent it three weeks before. That evening Jackson was arrested (被捕)for failing to pay his bill. He t station for fifteen days.

1. t_______ 2. w_______ 3. f________ 4. s__________ 5.n________

6.o______ 7. w________ 8. w________

八、书面表达

假如Mary 是上学期来到你们班学习的一个美国女孩,请根据以下内容,向你的朋友介绍她的一些学习情况,要求条理清楚、语句通顺,字数70左右。

1、她以前在美国的一所男女混合学校读书。

2、现在对汉语有浓厚的兴趣,但学习不够用功。

3、擅长数学,每天花大量时间做练习。

4、家政是她最爱的学科,可以学习如何自己做事情。

5、她是我们班学习最好的学生之一。当我们英语学习有困难时,我们会向她求助。

6、她说在这里学习很愉快。

_______________________________________________________________________________

_______________________________________________________________________________

_______________________________________________________________________________

_______________________________________________________________________________

______________________________________________________________________________________________________________________________________________________________

_______________________________________________________________________________

_______________________________________________________________________________

_______________________________________________________________________________

_______________________________________________________________________________

初二英语寒假作业(三)

一、选择填空:

( ) 1. I can‟t speak English _______ you do.

A as well as B as good as C as better as D so well as

( ) 2. Did he and you enjoy _______ at park?

A yourselves B themselves

C yourself and himself D himself and yourself

( ) 3. The third cake is ________ of all.

A delicious B more delicious C most delicious D the most delicious

( ) 4. Nancy‟s classroom and John‟s classroom are _______.

A like B same C alike D the same as

( ) 5. Simon scores _______ points.

A fewest B least C the fewest D the least

( ) 6. We ______ friends for almost 10 years.

A have made B have been C have become D have turned

( ) 7. We should share things with others and help people ______.

A in hospital B in need C in the hospital D in bed

( ) 8. I want to tell you about my ______ friend Betty.

A true B real C really D truly

( ) 9. She is kind and never says a bad word about _______.

A anyone B everyone C someone D nobody

( ) 10. He looked so tired _________ too much hard work.

A because B because it‟s C because there are D because of

( ) 11. You can take a taxi _______ the center old Beijing to learn more about old Beijing.

A in B along C circle D around

( ) 12. I am going to see the monument _______ the People‟s Heroes at Tian‟an men Square

A of B about C with D to

( ) 13. Do you know how many _______ a cow have?

A stomach A stomaches C stomachs D stomachies

( ) 14. I wonder where ________.

A is she hiding B does she hide C she does hide D she is hiding

( ) 15. Please don‟t tell him about it. OK, I_______.

A don‟t B won‟t C am not D didn‟t

( ) 16. She is very clever______ she is good at math.

A and B but C or D because

( ) 17. My sister loves playing softball, she ______ two hours _______ every day.

A spends, to practice playing B spends, practicing playing

C spends, practicing to play D takes, to practice to play

( ) 18. _________? He is friendly and polite.

A What does he look like B What does he like

C What is he like D What is he alike

( ) 19. A horse is _______ animal. It helps people a lot.

A a useful B an useful C a useless D an useless

( ) 20. People in Britain say “film” while people in American say “_______”.

A fall B hall C elevator D movie

( ) 21. —Is ______ here? —No, Li Lei and Wang Ping have asked for leave.

A somebody B anybody C nobody D everybody

( ) 22. He spent lots of money ______ clothes.

A buying B on C to buy D A and B

( ) 23. Where are you going ______ your winter holidays?

A in B at C for D of this afternoon.

( ) 24. We went to travel with some friends of _______.

A us B ours C our D ourselves

( ) 25. My uncle arrived ______ Shanghai ______ the next afternoon.

A at… in B in… on C in… on D in…/

( ) 26. Some boys think maths is ______ English. I think so.

A much difficult than B more difficult as

C less difficult than D so difficult as

( ) 27. On the long table ________ a glass of water and some nice food.

A is B are C have D has

( ) 28. What do you often do ______ the end of the week?

A on B by C at D in

( ) 29. China is a country _________ a long history.

A have B has C with D without

( ) 30.Yesterday she ______ a red coat and looked beautiful.

A wore B put on C put up D dressed

二、 用所给词的适当形式填空:

1. Giant Panda is a _______ (danger) animal because there are only about 1,000 in the world.

2. It is important for us ______ (know) more about nature.

3. Mr. Green advised them _______ (read) more during summer holidays.

4. Some _______ (hunt) are good at hunting animals.

5. I‟m sure everyone likes _______ (peace) life.

6. The boy ran to his mother ______ (wild).

7. The story happened on a ______(fog) morning.

8. If no one ______ (buy) animal fur, people ______ (not kill) wild animals.

9. How many ______(wolf) are there in the picture?

10. Who has the ______ (key) to the doors of the classroom?

三. 完型填空:

Wang Fang is 16 years old. She helped ____1____ neighbor out ___2__ a fire. ___3__ 10th May, Wang Fang was at home alone. Suddenly, she heard someone __4___ “Fire! Fire!” She ran outside. She saw a lot of smoke from next door. She went __5___ and saw the ___6___ Mr. Sun, in the kitchen. He could not get out because he hurt his leg.

Wang Fang quickly ran back to her flat and __7___ water over her jacket. __8___ she rushed into Mr. Sun‟s kitchen ___9__ him. At last she put out the fire ___10___ a blanket and helped Mr. Sun out.

( ) 1. A her B she‟s C herself D herself‟s

( ) 2. A from B form C of D on

( ) 3. A In B At C On D From

( ) 4. A shout B shouting C to shout D shouted

( ) 5. A in B into C out D outside

( ) 6. A 79 years old B 79-years-old C 79-year old D 79-year- old

( ) 7. A pulled B poured C pushed D threw

( ) 8. A First B Next C After D Then

( ) 9. A help B to help C to save D and save

( ) 10. A use B using C used D with

四. 阅读理解:

A

The children in Miss Ling‟s class are not like most children. They don‟t watch TV. Miss Ling asked her class not to watch TV for one week. She told them that they could have more fun doing other things. The children said, “Let‟s find out if Miss Ling is right.” Instead of watching TV, they read, made things and played games with families.

The children had so much fun that they are not going to watch TV for a year. Would you like to stop watching TV?

( ) 1. The best title is ______.

A A Good TV Show B Stop To Watch TV

C Children Have Fun Without TV D Miss Ling Doesn‟t Buy Any TVs

( ) 2. Miss Ling asked her class not to ______.

A sleep B eat C read books D watch TV

( ) 3. The story says that the children _________.

A played games B went swimming C grew plants D watched TV

( ) 4. Miss Ling is a _________.

A doctor B teacher C farmer D child

( ) 5. How long are the children not going to watch TV?

A For twelve months B For 365 days

C For half a year D For three years

B

Bats are the only flying mammals(哺乳动物) in the world. They cannot see very well, so people believe they have no trouble flying no darkest nights and finding their way around very well. How can bats fly and see at night? They fly by radar(雷达).

The bat‟s radar system(系统) works the same as ships and planes do. As a bat flies through the air, he makes a sound. The sound is high enough for his message. In this way it tells the bat where the thing is.

Bats go out to look for food only at night. In the daytime they rest in some dark places.

( ) 1. The bat is ______ in Chinese.

A 蝙蝠 B 老鼠 C 蚊子 D 蜻蜓

( ) 2. The bat has no trouble flying on the darkest nights because ______.

A its eyes are good B it can‟t see well

C it can make a sound D it has a kind of radar system to help it

( ) 3. When does the bat begin to go out to look for food?

A In the daytime. B In the early morning.

C When night comes. D In the late afternoon.

( ) 4. The bat “sees” things with its _________.

A eyes B nose C mouse D ears

( ) 5. Which of the following is right?

A The bat is the only flying animal in the world.

B The bat is blind, but it can “see” its food.

C Not all bats are afraid of flying in the sun.

D The bat often comes out during daytime, too.

五. 单句改错:

1. Don‟t worry. There is little time left.

2. Because I can‟t see anything, so I never go out at night.

3. There is going to have a concert next Sunday.

4. What a beautiful weather it is today!

5. Tom and Jack are all students.

6. Which is much interesting, Shanghai or Beijing?

7. How long do you go home? Once a week.

8. Your mother must keep health.

9. What is the weather like yesterday?

10. The floor is all wet. Please mop up it.

11. They have more rains this year than last year.

12. She crashed her car on a wall.

13. He was died in a car accident.

14. A boy fell down a tree and broke his left leg.

15. Look! His clothes are on a fire.

六. 完成句子;

1. 吴先生邀请我参加他们学校去世界公园的旅游。

Mr. Wu invited me _____ _____ _____ their school trip to the World Park.

2. 利奥的铅笔盒和克丽斯廷的铅笔盒尺码一样大小。

Leo‟s pencil box is ______ _____ _____ as Christine‟s pencil box.

3. 朱莉同意和他们去。

Julie agrees _____ _____ ____ them.

4. 长大后,贝蒂想当歌手。

Betty wants to be a singer when ______ ______ ______.

5. 她明亮微笑的双眼令她看起来真的很漂亮。

Her bright, smiling eyes _______ her ______ _______ pretty.

____________ ____________ ____________ ____________ ____________ ____________ ____________ ____________ ____________ ____________ ____________ ____________ ____________ ____________ ____________

初二英语寒假作业(四)

一、单项选择

( )1. It‟s necessary ______ us ______ read English every day.

A. of, to B. for, to C. for, for D. to, to

( )2. She likes _____ her joy ______ her friends.

A. share, with B. sharing, with C. share, for D. sharing, for

( )3. Do you think British students spend___ time ___ their homework than Chinese students?

A. less, doing B. less, to do C. fewer, doing D. fewer, to do

( )4. It was getting dark. There were ______ people in the street.

A. less and less B. fewer and fewer C. less and fewer D. fewer and less

( )5. He finds _______ to make a paper ship.

A. it easy B. that easy C. it easily D. that easily

( )6. We have got everything ready. So we need _______.

A. something else B. else something C.nothing else D. else nothing

( )7. The teacher told us to leave school early ______ the snow got worse.

A. but B. so C. because D. and

( )8. If Jim ______ free tomorrow, I‟ll ask Tom instead.

A. isn‟t B. won‟t be C. won‟t D. doesn‟t

( )9. ----Did you buy the camera yesterday?

----No. I didn‟t buy it ____ my aunt would give me one.

A. until B. because C. if D. before

( )10. The teacher is coming. Stop ______ and keep ______.

A. to talk, quiet B. talking, quiet C. to talk, quietly D. talking, quietly

( )11. Our government provided food and money____ the people in the disaster areas.

A. for B. with C. to D. on

( )12. There will be a strong ______ tomorrow and it will be quite ______.

A. wind, wind B wind, windy C. windy, wind D. windy, windy

( )13. The news report is about ______ a month ago.

A. what he happened B. what he happened to

C. what happened him D. what happened to him

( )14. The capital of the USA is_______.

A. London B. New York C. Bangkok D. Washington DC

( )15. --- My brother fell off his bike and hurt his legs. --- ______.

A. That‟s too bad B. He is too careless

C. He should be careful D. I am sorry to hear that

( )16. The land is _______ after a heavy rain.

A. much weter B. much wetter C. more weter D. more wetter

( )17. The sunlight came in ______ the window.

A. through B. across C. in D. with

( )18. What did you do in the garden? I watched John ______ his bike.

A. to mend B. mended C. mend D. mends

( )19. The big box is ______ heavy and I can‟t carry it.

A. much too B. too much C. so much D. very too

( )20. Daniel thinks climbing is ______ diving.

A. so excited as B. as excited as C. so exciting as D. as exciting as

二、完型填空

It was December 25, 1972. Marie, a 13-year-old Australian girl, was very happy. It was and Marie‟s mother made an unusual cake. She put four small coins(硬币) . mother didn‟t notice that.

. She coughed and couldn‟t speak. Six weeks later she felt went to the hospital.

(X光). Marie‟s mother asked the doctors, “Why can‟t Mariedoctor said, “We don‟t know. Maybe she will speak again. Maybe she won‟t. We‟re, but we can‟t help her.”

For twelve years Marie didn‟t speak. SheOne day when Marie was 25 years old, she got ill. She began to cough. She coughed up a lot of blood(血) small and black from her throat(喉咙). What was it? Marie didn‟t know. She took it to the hospital. A doctor said, “This is a coin.”

The doctor told Marie, “I

( )1. A. party B. Christmas C. holiday D. winter

( )2. A. the cake B. her pocket C. Marie‟s bag D. the box

( )3. A. friends B. health C. luck D. life

( )4. A. teacher B. brother C. daughter D. family

( )5. A. began B. finished C. did D. made

( )6. A. paid B. found C. missing D. put away

( )7. A. tired B. home C. back D. ill

( )8. A. worse B. terrible C. better D. lonely

( )9. A. still B. also C. yet D. often

( )10. A. looked for B. looked over C. looked after D. looked up

( )11. A. speak B. listen C. tell D. hear

( )12. A. sure B. busy C. worried D. sorry

( )13. A. put up B. sent up C. grew up D. got up

( )14. A. something B. nothing C. anything D. everything

( )15. A. sure B. tell C. ask D. think

三、 阅读理解

(A)

Everyone needs friends. There is an old saying: Friends are God‟s way of taking care of us. But how do you find real friendship and keep it?

“The Care and Keeping of Friends” by American author Sally Seamans tells young students some smart ways to find friends. Sally says finding friendship is just like planting a tree. You plant the seed and take care of it to make it grow.

First, you should choose a friend. What makes a good friends? It is not because a person has money or good looks. A good friend should be kind and patient. For example, if you have a bad day, a good friend should listen to your complaints(抱怨) and do their best to help. To make a friend, you cannot be too shy. You should make each other happy and share your lives.

But things cannot always be happy. Even the best friends have fights. What should you do when you have a fight with your friend? You have to talk to them. When there is no one around, have an honest talk. If he or she doesn‟t want to talk, you could write a letter.

Sally says there are three steps to being friends again: Tell him or her how you are feeling; say what your friend has done wrong, and explain why you did this or that. The book also has advice on some small but important things like celebrating your friends‟ success. Even if you haven‟t had a real friend before, you will start to think of having one if you read this book. Because the book tells that friendship is the most important thing in your life.

( )1. The writer of the text really wants to tell us________.

A. life is great if we have friends B. an old saying

C. Sally Seamans is an American writer D. God‟s way of taking care of us

( )2. “The Care and Keeping of Friends” is_______.

A. the word of the God B. a way of making good friends

C. the name of a book by Sally D. a story from a newspaper

( )3. A good friend should_______.

A. have a lot of money B. have good look

C. be kind and patient D. listen to you

( )4. Good friends will_______.

A. always be happy B. do their best to help you

C. never do wrong D. not have a fight

( )5. What can‟t you do if your good friend has done wrong?

A. Write a letter to him. B. Have an honest talk with him.

C. Tell him or her your feeling. D. Have a fight with him.

(B)

Can you make animals work for us? Some scientists think that one day we can teach animals to do a lot of things for people. In a film shown on TV, you may see elephants, monkeys, tigers or some other animals are always given a little food to eat after they have done something. Scientists say that people can teach many different animals to do some of the easy work if they know they will get something to eat. We all know elephants can carry large logs(圆木),and dogs can look after houses.

And we even teach animals to work in factories. In America, for example, people have used apes to help make cars and scientists think that those large monkeys may drive trains one day. Now people are trying to get them to do the same thing that man does.

( )6.Scientists think animals can help people to do something .

A. if they know they will get food after doing something

B. because animals are so clever.

C. if they know the work is easy enough

D. because most animals are friendly to man

( can do some heavy work because they are so strong.

A. Tigers B. Elephants C. Monkeys D. Dogs

( )8.As we know, dogs can our houses.

A. build B. clean C. look after D. look at

( )9.Scientists are planning to make big monkeys to .

A. make trains B. carry logs C. drive trains D. work at school

( )10. If we teach animals carefully, .

A. some animals can do the same things as a man does

B. all the animals can do some work for us

C. many animals can do all the work as we do

D. we may get many animals work without giving them food

四、词形变换

1. Don‟t forget _______ (lock) the door when you leave home.

2. Do you know what________(do)next?

3. The new tie makes him _______ (look) smart.

4. I will give it to her as soon as she ____(come) back next week..

5. Our teacher told us _______ (not get) to school late next time.

6. Do you mind ______(close) the window?

7. If you want to learn English better, you should practise _______ (speak) English often.

8. Both the twins enjoyed ______(they) at the party.

9. My pet lives_______(comfortable)in his cage.

10. Tim was ________ (trap) in a dark place when the earthquake stopped.

五、完成句子

1.你要保守秘密,否则这会使他不高兴。

You should _______ .

2.今天完成这项工作是不可能的。

It is __________ _________ __________ the work today.

3.我们应该鼓励孩子们保护动物。

We should __________ children __________ animals.

4. 他没有你强壮。

He isn‟t_____ ______ _____ you.

5.我们需要更多人来认识到湿地的重要性,并且采取措施来保护野生生物。

We need more people to know the _____of the wetlands and___ ___to protect wild animals.

六 、书面表达

请根据下列内容,写一篇短文,要求要点全部写出。

昨夜,一场可怕的洪水袭击了我村,冲走了许多房屋。洪水到来时我和我父母仍在睡觉。我们急忙到屋外,看到四周都是水。我们将餐桌从屋内移到了屋外,但它的大小只够两个人坐在上面。屋前有颗大树,于是我快速爬上了那颗树并在上面待了好几个小时。

___________________________________________________________________________________________________________________________________________________________________________________________________________________________________________________________________________________

初二英语寒假作业(五)

一、单项选择

( )1. He never tells lies, so he is _____ honest boy. It‟s not good to be _____ dishonest one.

A. a , an B. an , a C. /, a D. an ,the

( )2. When his father London, he went to see the Big Ben.

A. arrived B. reached C. got D. arrived to

( )3. Let‟s enjoy the song “Yesterday once more”. It sounds

A. bad B. sadly C. well D. wonderful

( )4. __________ nature reserves _________ us learn more about protecting animals.

A. Studying , helps B Studies , to help C. Study , helping D. Studying, help

( )5. Red-crowned crane is a kind of bird.

A. endanger B. danger C. endangered D. dangerous

( )6. I am in places of in Nanjing.

A. interesting, interest B. interested, interest

C. interested, interesting D. interesting, interested

( )7. “X plus Y is more than Fifteen percent.” This sentence can also write as “_________”

A. X+ Y>15% B. X+Y≥15% C. X+Y=15% D. X+Y<15%

( )8. The floor is quite wet, my teacher asked us _________ quickly.

A. to mop up it B. mopped it up C. to mop it up D. mop it up

( )9. If we keep the land, giant pandas will have to live.

A. to take, anywhere B. taking, anywhere C. to take, nowhere D. taking, nowhere

( )10. If someone shows good manners to others, he is a ________ person.

A. impolite B. polite C. rude D. dispolite

( )11. He wears small, round glasses and they make him _________ smart.

A. to look B. to be C. looks D. look

( )12. He was all wet when he got home, because he his umbrella in the underground.

A. took B. forgot C. left D. brought

( )13. The number of cranes in the world is getting these years.

A. less B. much C. little D. smaller

( )14. The Taiwan Earthquake killed people.

A. two thousands B. two thousands of

C. thousand of D. thousands of

( )15. Which of the following is not a natural disaster?

A. A car crashed into the tree B. A big flood

C. A storm D. The Taiwan earthquake

二、完形填空 lovely birds. will soon catch it.

Seagulls swim well, but they don‟t often dive for fish. They are also good at flying _____said she didn‟t like the noise they made.

( ) 1. A. watch B. to watch C. watching D. watched

( ) 2. A. understood B. learned C. found D. thought

( ) 3. A. throw B. take C. send D. pull

( ) 4. A. with B. to C. for D. of

( ) 5. A. below B. above C. on D. in

( ) 6. A. to come B. came C. coming D. comes

( ) 7. A. help B. beat C. put D. kept

( ) 8. A. hit B. arrived C. got D. touched

( ) 9. A. water B. land C. sky D. ground

( ) 10. A. eyes B. feet C. legs D. wings

三、阅读理解

(A)

Many animals do strange things before an earthquake. This news may be important. Earthquakes can kill people and knock down homes. The animals may help to save lives.

Some animals make a lot of noise before an earthquake. Farmers have told about this. Dogs that are usually quiet have started to bark. Horses on farms have run around in circles. Mice have left their holes and run away. Cows have given less milk.

In a town in Italy, cats raced down the street in a group. That happened only a few hours before an earthquake. In San Francisco, a man kept tiny pet frogs. One Sunday, the frogs jumped around more than ever. They make loud noises, like bigger frogs. That night, an earthquake struck the city.

People want to know when an earthquake is coming. Then they could get away safely. Right now, there is no sure way to know ahead

of time. Maybe the best idea is watching the animals.

( )1. This passage is mostly about .

A. how animals act before an earthquake B. how an earthquake starts.

C. how mice leave their homes. D. how animals live every day

( )2. Before an earthquake, quiet dogs A. ran away B. started to bark

C. climbed trees D. jump into the water

( )3. Before an earthquake, the frogs .

A. sang B. jumped into the water

C. left their homes D. jumped around a lot

( )4. There have been earthquakes in A. most countries B. Taiwan, China

C. Italy and San Francisco D. Chicago and Spain

( )5. People want to A. be in earthquake B. find out early about an earthquake

C. run around in circles D. follow the animals to run quickly

(B)

Young people can have problems with their minds. Some students become worried because they have to study very hard. Others have trouble getting on well with people like their parents and classmates.

Lu Jian, a student from Henan, could not understand his teacher and was doing badly in his lessons. He became so worried about so that he started to cut his finger with a knife.

Another student, 16—year—old Liu Fang from Guang dong, was afraid of exams. She got very worried in one, and when she looked at the exam paper. She couldn‟t think of anything to write.

A recent report from China Daily says about 14% of Beijing teenagers have mental problems. Their troubles include worried and very unhappy. And having problems in learning and getting on with people. Many students who have problems won‟t go for advice or help. Some think they will look stupid if they go to see a doctor. Others don‟t want to talk about their secret.

Gao Xiao, an expert(专家) on teenagers from Beijing Anzhen Hospital has the following advice for teenagers:

*Talk to your parents or teachers often.

*Take part in group activities and play sports.

*Go to see a doctor if you feel unhappy or unwell.

( ) 6. The students who often become worried or have trouble getting on with others may have _________________.

A. mental problems B. a headache C. knives with them D. no parents

( ) 7. Lu Jian cut his finger with a knife because______________.

A. he was afraid of his teacher. B. he wanted to frighten his parents

C. he was so worried about his studies D. his finger was badly hurt

( ) 8. Liu Fang‟s problems happened when __________________.

A. she studied very hard B. she had exams

C. she talked with her parents D. she thought of something

( ) 9. Students who have problems won‟t ask others for help because _______________.

A. they won‟t let others think they are stupid

B. they don‟t think doctors can help them.

C. they don‟t want to tell their secret to others

D. both A and C.

( ) 10. Gao Xiao‟s advice tell us that ___________________.

A. it‟s better for the students who have mental problems to join others

B. it‟s unnecessary for them to be with others.

C. only group activities and sports can help them.

D. teachers and parents can‟t do anything about mental problems

四、填空与改错

A)根据括号里所给的汉语、英文解释、首字母及句意填入合适的单词

1. Let‟s go bird watching in the (森林).

2. The girl __________ (尖叫) when she saw the snake.

3. Is it

5. Do you know the 原因) why he didn‟t come to your party?

B)根据句子意思,用括号所给单词的适当形式填空。

1. It is _________ (correct) to hunt red-crowned cranes.

2. If you still do your homework _________(care), you will lose the exam again.

3. The members of our club encourage hunters_________(not kill) rare animals.

4. If it 5. Look! The bus __________ (come).

C)下列各句的A、B、C、D中各有一处错误,在不改变句子意思的前提下,将正确答案写在题后的横线上。

(A) (B) (C) (D) (A) (B) (C) (D) tomorrow.”

(A) (B) (C) (D)

4. Don‟t make so (A) (B) (C) (D) (A) (B) (C) (D) D

2. you have enough food when you go camping.

3. Last time, after the big earthquake, Sandy ______________.

4. Is your pet dog good at 5. If you play football in the busy street, you _________________.

五、句型转换

1. People were frightened to look at each other. (同义句)

People looked at each other _____________.

2. It‟s important for us to read English every day. (同义句)

We must know English every day.

3. Mr. Wu warned him: “Don‟t run in the street.”(同义句)

Mr. Wu warned him .

4. It‟s 划线提问) the weather today?

划线提问)

六、书面表达

A)翻译

1. 狐狸有好的视觉,听觉和嗅觉。

Foxes have 2. 人们改变湿地来为农场腾出更多空间。

People changed the wetlands farms.

3. 如果你每天练习,你就将很好听地唱歌。

If you practice every day, you _____________ songs _______.

4. 人们逃向四面八方。 People run .

5. 当我听见一个巨大的声响,一阵恐惧涌上心头。

B)周末你去北京野生动物园参观。请根据你所观所感,写一篇短文,要点如下:

1. 有很多游客,他们给猴子喂食,和老虎玩耍。

2. 动物和人很友好,我们应该善待动物。

3. 保护环境非常重要,动物园内不要随地扔垃圾。

4. 知道了野生动物保护的重要性。因为保护动物就是保护我们自己。

____________________________________________________________________________________________________________________________________________________________________________________________________________________________

初二英语寒假作业(六)

一、选择填空

( )1. Which is the cheapest?

( )2. My father was born_______1964. He is forty years old.

A. at B. on C. in D. to

( )3. Animal World is the most interesting TV________.

A. play B. sport C. team D. program

( )4. Please give________ a slice of bread, Mom. I‟m very hungry.

A. I B. my C. me D. myself

( )5. Paul likes to help others. He is very________.

A. happy B. kind C. quiet D. funny

( )6. — ________ water do we need? — A bottle.

A. How many B. How long C. How often D. How much

( )7. My mother _________lots of things and took the subway back home yesterday.

A. buys B. bought C. is buying D. is going to buy

( )8. — Could you please go sightseeing with us this Sunday?

— _________. I have to finish my homework.

A. Yes, sure B. I‟m sorry C. Excuse me D. Good idea

( )9. I don‟t like the weather in Xi‟an. It‟s dry in summer. It______ rains.

A. seldom B. usually C. always D. often

( )10. We‟re going to the Circle Theater in John‟s car. You can come with us _______ you can meet us there later.

A. but B. and C. or D. then

二、完形填空。

( )11. game B. video C. day D. sale

( )12. put B. entered C. jumped D. stayed

( )13. played B. did C. bought D. took

( )14. A. in the room B. in the yard C. at school D. at the cinema

( )15. A. exciting B. interesting C. relaxing D. boring

(B)

What is a museum? A museum is a good place to keep and beautiful things. A museum may be a place to learn about science. A museum can be a place to the history about art or Indians or animals.

What is inside a museum? Some museums have old cars and airplanes. Many museums have pictures and statues. and old bones. One museum even has a coal mine inside!

Many cities have museums. Some very small towns have museums, too. Indianapolis has a free museum for children. Children do not have to to get in. They like to look at the dinosaur bones(恐龙化石). They see a white bear ten feet tall there.

On Saturdays, Does your town or city have a museum?

( )16. A. new B. old C. bad D. fun

( )17. help out B. hang out C. try out D. find out

( )18. Others B. The others C. The other D. Another

( )19. go B. like C. pay D. learn

( )20. boys B. girls C. children D. parents

三、阅读理解

(A)

Joe 正在介绍自己的像册。请你为每一幅照片划到相应的描述,并将字母填入相应的( )内。

Hi! My name is Joe Williams. I‟

m 15 and I come from Atlanta. This is my photo album.

A. These are my grandparents, Timothy and Mary. They come over to our house almost every Sunday for dinner, and they always help with the cooking. My grandpa was a teacher, but now he‟s retired. He helps me study when I have exams. My grandma loves playing with my little sister.

B. Here I‟m with my friends. We‟re in the came class and we love basketball. I‟m tall, so I play on the school team. My favorite team is the Chicago Bulls.

C. After school I usually have lots of homework and very little free time. My favorite hobby is playing the guitar. And I also play in the school band. We aren‟t very good, but we keep practicing.

D. These are my parents, Alex and Beth, and my little sister Jenny. She‟s only five. She loves picnics in the park because she can run around and she often feeds animals. She loves animals very much. She says she wants to be a doctor for animals when she grows up.

(B)

Jenny mixed up the letters with the replies. Read the three letters and the replies A~C, match each letter with the correct reply. 你能将Jenny的回信交给不同的来信者吗?请将字母填入相应的( )内。

( )28. What is Mary‟s problem?

A. She did badly in some important exams.

B. She worries too much about her exams.

C. She can‟t do her homework very well.

( )29. What shouldn‟t Mary do?

A. Find more time to relax than before.

B. Do a lot of exercises and think positive.

C. Drink coffee or tea before going to bed.

( )30. Why don‟t Nina‟s parents want to buy her a mobile phone?

A. It is too expensive and they can‟t pay for it.

B. It isn‟t very good for Nina‟s health.

C. They‟re afraid Nina will use it a lot.

( )31. If Nina gets a mobile phone, she should __________.

A. pay the bills all by herself

B. ask her parents to pay the bills

C. ask her parents for more pocket money

( )32.Why doesn‟t Jim spend much time with his father?

A. Because Jim has a lot of homework to do.

B. Because Jim‟s father works a lot every day.

C. Because Jim‟s father got a new job in another city.

(C )

阅读短文后选择正确答案.

As teenagers, you have many dreams .These dreams can be very big, such as winning the Nobel Prize, or they can be small. You may just want to become one of the ten best students in your class.

Once you find a dream, what do you do with

it? Do you ever try to make your dream real?

“Follow Your Heart”by Australian writer

Andrew Matthews tells us that making our dreams

real is life‟ You may think you‟re not very good at some

school subjects ,or that it is impossible for you to

become a writer. These kinds of thoughts stop you

from getting your dream, the book says.

In fact, everyone can make his dream come true.

The first thing you must do is to remember what your

dream is.

Don‟t let it leave your heart. Keep telling

yourself what you want. Do this step by step and

your dream will come true faster because a big dream is,

in fact, many small dreams.

You must also never give up your dream.

There will be difficulties on the road to your

dreams. But the biggest difficulty comes from yourself. You need to decide what is the most important. Studying instead of watching TV will lead to better exam results, while saving five yuan instead of buying an ice cream means you can buy a new book.

As you get closer to your dream, it may change a little. This is good as you have the chance to learn more skills and find new interests. ( )33.“Follow Your Heart”is .

A. the first thing you must do to make your dream real

B. the most important dream to have for teenagers

C. the name of a book by Andrew Matthews

D. the name of a famous Australian writer

( )34.If you want to make your dream real, you should .

A. remember what you dream is first

B. keep telling yourself what you want

C. never be afraid of the big difficulties

D. try to start with all the things above

( )35. From the text, we know the word challenge means .

A.变化 B.挑战 C. 决定 D. 态度 ( )36. What‟s the best title for the text?

A. Keep your dream in your mind. B. How to find your real dream C. Never stop dreaming of success. D. Teenagers have many dreams.

四、语言综合能力运用

一、很多同学都有自己的特长,如果有机会你将在哪里展示这些特长?你将如何展示这些特长呢? (A) 请根据提示,将所给单词、词组分类;只写字母即可。

1. Our talents are ___, ____,____,_____,_____.

2. We‟ll show our talents in/on / at the ____,_____,____,____,_____,_____. 3. We can use the ____,____,______,_____,_____ to show our talents. (B)根据自己的实际情况,选择一项内容填写。 1. If you have a talent, how do you show your talent?

I am good at , I‟ll To show my talent.

2. If you don‟t have a talent, what are you going to do?

I am not good at

二、周末就要到了,让我们为周末做一些准备吧!

(A)Nancy的父母打算周末邀请朋友共进晚餐,妈妈想让Nancy帮助做一些准备,请根据图片补全文段。 (4分)

Nancy,

I‟? ‟ Thanks Mom

(B)本周日你邀请朋友参加你的生日聚会。你想请妈妈为你做哪些准备 — 烹制美味食品? 买些饮料、小吃?为喜欢音乐的朋友准备些CD?还要买生日蜡烛(candles)吗?

你可以依照上面的文段,给母亲留言,写出你的请求。(40~50words) Dear Mom,

It will be my birthday this Sunday. I invited my friends to take part in my birthday party. .

三、寒假就要到了,你父母想带你外出旅游。请根据下表提供的信息,在三个旅游地中,选择一个你最想去的地方和合适的交通方式,写出一篇旅游计划,并简单说明理由。

初二英语寒假作业(七)

一、单项填空

1. look! The traffic light is red. You __________ stop. A. must

B. may

C. can

2. —What does the sign mean?

—It tells us ___________ on the road.

A. to ride B. not to ride C. don‟t ride 3. —The bus is too __________ to get on. Shall we wait for the next one? —All right.

A. clean B. empty C. crowded 4. —A stranger came to my room ___________ I was sleeping last night. —Really! Were you scared(恐惧的)?

A. while B. after C. before

5. —Will the girl keep the smallest box of chocolates for _____________? —I guess she will choose it, for she is very kind. A. her B. herself 6. —Did your father buy you a birthday present?

C. yourself

—Yes. He bought me a toy elephant, _____________ I don‟t really like it. A. and B. so C. but 7. —Will there be ___________ fine days in Beijing next year? —Yes. The sky will become clearer.

A. fewer B. more 8. —Wow! So many people. Let‟s go to the front. A. jump, will be 9. —Angry? What‟s up?

B. jump, are

C. less

—Oh, no. If you __________ the queue, other people ____________ unhappy.

C. will jump; will be

—Yes. Misha ____________ my mobile phone without telling me. A. took back B. took off 10. —Maybe you can take a taxi to the bank. It‟s faster. —_______________ A. Thanks. I will. B. Yes, please.

C. Good idea. Let‟s hurry.

11. _________ her mother ____________ her father lives with the little girl. They go to work in a big city. A. Both, and B. Either, or 12. —Look! Lin Tao is catching up. ________________

C. Neither, nor

C. took away

—Come on! Lin Tao. A. Is he running fast? B. How fast he is running!

C. How fast is he running?

13. —What‟s wrong, Jenny? You look unhappy.

—Betty has the same clothes ____________ I do. I want to be different. A. as B. with C. to 14. —I‟m ___________ the dog downstairs. It often makes too much noise. —Why don‟t you talk to your neighbour, the dog‟s owner? A. surprised at B. worried about 15. —What ____________ at nine o‟clock last night, sir? —Why? What happened? A. were you doing

二、完形填空

Elena was not a good student. Her head was _____16_____ most of the time. She wanted to listen in class and to study, but other things were more important: her boyfriend, her clothes, her hairstyle, and _____17_____. Every time she tried to concentrate(集中)on her lesson, her mind was gone.

One day, her English teacher, Mrs. Frederickson, told the students there was going to be an important test. She reviewed all week. Where was Elena? She _____18_____ to be there, but her mind was not.

“Did you copy the questions? Did you do the exercises? Did you write your homework? Elena, _____19_____! You‟re going to fail(not pass)the test,”warned Mrs. Frederickson.

The day of the test arrived. Elena didn‟t study very much, and she didn‟t know _____20_____ questions. She tried to guess but soon stopped.

Next to Elena sat Ivan, a very _____21_____ student. Elena didn‟t want to fail the test, so she decided to cheat. She knew it was wrong, but she had no _____22_____ of passing the test. She began to copy all of Ivan‟s answers.

Ivan noticed Elena cheating and was very _____23_____. Quickly he changed all his answers so that they were not correct. So did Elena. Then, _____24_____ his paper, he quickly changed his answers back to the way they _____25_____. Elena wasn‟t quick enough and the bell rang. Ivan turned to Elena and laughed.

“Honesty(诚实)is the best policy,”he said. “Now all your answers are wrong for sure.” 16. A. in use 18. A. seemed 20. A. some 21. A. helpful 22. A. hope 23. A. pleased 24. A. taking 25. A. did

B. in the clouds B. subjects B. tried B. wake up B. any B. key

B. polite B. angry B. passing B. might

C. in the dark C. television C. wanted C. make up C. much C. serious C. idea C. sad

C. hiding C. could

D. in the past D. school D. liked D. look up D. many D. careless D. wish D. interested D. holding D. were

17. A. exercises 19. A. come up

B. did you do

C. are you doing

C. fed up with

三. 阅读理解

A

Dan lives in Maine. He has very poor eyesight(视力)for years. He now has a guide animal to help him. The guide animal is called Cuddles. Cuddles is really a small horse. She is fully grown and just 22 inches tall. Cuddles gives Dan the help he needs and also gives him love and comfort(安慰).

Dan first heard about such horses on a television show, He found out about two horse lovers named Don and Janet Burleson. They train small horses to lead(引导)blind people.

The Burlesons got the idea for guide horses during a trip to New York City. While riding in a carriage(马车)pulled by large horses, they thought about their small pet horse Twinkie. She was friendly, clever, and kind. They wanted to know if they could train Twinkie to help a blind person. When the Burlesons returned home, they began to train Twinkie, like training dogs. Twinkie learned very well, so they decided to train other little horses.

Now the Burlesons start training these special horses when they are six months old. The training takes about a year. Each horse has a lot to learn during that time, such as to lead a blind person safely, to ride in a car and on a bus, to climb stairs and to ride in lifts. These are not places where a horse usually goes. It takes time and practice for the horse to get used to being inside buildings.

26. Dan has ____________ to help him. A. a dog C. Don

B. a horse D. Janet

B. One year.

27. How long does it take to train a guide horse? A. Six months. C. Two years.

D. Six years.

B. trained dogs

D. moved to New York City

28. Before the Burlesons started training many horses, they ___________. A. trained Twinkie C. visited Dan

29. Look at the chart below.

Which one can you put in the empty box? A. Eat grass

B. Have good eyesight C. Live 30 years D. Climb stairs

B

I met a friend yesterday but had forgotten his name. I know that he had forgotten mine, too. I felt very embarrassed(尴尬的)at that time and didn‟t know what to do. My question is: What would you do if you met this embarrassing situation(情况)?

__________________________________________________________________________

Bring in a third person. Pretend(假装)you know that person. The third person will(with luck)be too polite to say that he has never met you. You say to the third person, “I‟m sorry, but I‟ve forgotten your name.”This person will then introduce himself. You can then introduce yourself. Your friend will now introduce himself. Now you have got your friend‟s name.

Jane,

Lincoln, England

--------------------------------------------------------------------------------------------------------------------- Move to France. “Bonjour monsieur / madame”is OK if you don‟t know someone really well.

Joseph,

London, England

---------------------------------------------------------------------------------------------------------------------

The orchestra conductor(交响乐指挥家)Sir Thomas Beecham told this story:After conducting a concert, a woman started to talk with him, but he didn‟t know her. When she talked about her brother, Beecham saw his chance. He asked about her brother‟s health and asked if he was still doing the same job. “Oh, yes,”she said, “he is very well and is still king.”

Mark,

Oxford, England

---------------------------------------------------------------------------------------------------------------------

You could try the Samoan(萨摩亚人)method. There, people say hello to one another with their own names. So Dick Collins would say in Samoa, “Hi, Dick Collins!”And Jonathan Green would answer,“Hi, Jonathan Green.”

Mike,

County Cork, Ireland

30. What would Jane do if she met this situation?

A. She would pretend to know her friend and talk with him. B. She would get her friend‟s name by bringing in a third person. C. Jane thinks it‟s OK to ask someone‟s name straight. D. She would let a third person ask her friend‟s name.

31. Why did Sir Thomas Beecham ask something about the woman‟s brother?

A. Because he wanted to know how her brother was. B. Because her brother was one of his good friends. C. Because he wanted to know who she was. D. Because her brother was king.

32. Which of the following sentences is correct?

A. The report tells us about all kinds of embarrassing situations. B. The passage is most probably from a letter. C. It‟s very important to remember names in France. D. Sir Thomas Beecham spoke to the sister of a king.

C

Oseola McCarty left school in the sixth grade to go to work. For most of her 87 years, she took in wash from people of Hattiesburg, Mississippi. She never married, never had children and never learned to drive. All she ever had was work, which she saw as a blessing. Too many other black people in rural(乡下的)Mississippi didn‟t have even that.

McCarty spent almost nothing, living in her old family home, and tying her old Bible(圣经)with tape. Over tens of years, her savings-mostly dollar bills and change-grew to over $ 150,000. “More than I could ever use,”McCarty said.

So she is giving her money away to black students at the University of Southern Mississippi, where students have to pay about $ 2400 a year. “I wanted to share my savings with the children,”said McCarty, whose only regret(遗憾)is that she never went back to school. “I believed the money would do them a lot than it would me.”

The college has already awarded(奖励)$ 1000 in McCarty‟s name to Hattiesburg honors (荣誉)student Stephanie Bullock, 19. Though Bullock was in her fourth year and had high grades, her family couldn‟t afford four years of college. Then she learned that the university was giving her $ 1000 in McCarty‟s name. “It was a surprise and an honor,”Bullock said.

Bullock visited McCarty to thank her personally. Now she often visits McCarty, and drives her around. That fills a space in the woman‟s home that has been empty for years.

While McCarty does not want a building or a statue(塑像)in her honor, she would like one thing: to be at the graduation(毕业)of a student who made it through college because of her present. 33. The underlined sentence means ____________. A. she cut the toes off to fit the shoes B. she wore the shoes out to fit her feet C. she made the shoes right for her feet D. she threw away the thoes to buy right ones 34. From the passage, we know _________________. A. All the black students need money.

B. McCarty would like to be at students‟ graduation. C. Stephanie Bullock is good at her subjects. D. Bullock‟s visits disturbed McCarty‟s quiet life. 35. Why does McCarty give away her money? A. She got less education and wants to be helpful. B. She has no family and lives by herself. C. She made and saved a lot of money. D. She wants people to remember her forever.

四. 口语交际

从B栏中选出恰当的句子与A栏中的句子组成对话。

A

Patient:My head hurts badly, Doctor.

Doctor:Open your mouth and say“Ahh”. Hm„ _____36_____ Patient:Yes, but my temperature is all right. _____37_____

Doctor:You‟ve got a cold. _____38_____ Take this medicine twice a day and drink more water. Patient:____39_____

Doctor:It doesn‟t make any difference. Patient:Thank you.

Doctor:_____40_____ You‟ll be better soon if you can sleep well. Patient:Thanks a lot.

B

A. Nothing serious.

B. Shall I take it before of after meal?

C. Did you take your temperature at home?

D. What‟s wrong with me?

E. That‟s all right.

五. 语言知识运用

(一)请用所给词和词组的正确形式填空

41. —I would like to buy a pair of shoes for my son.

—What ____________ do you want?

42. —Could you help me ______________ this big wooden box? It‟s in the middle of the road.

—It‟s my pleasure.

43. —April Fool‟s Day is time to _______________________ others, isn‟t it?

—Yes. But remember not to hurt other people.

44. —What happened to Roy?

—He was in trouble but he _______________ all help yesterday.

45. After a long cold walk I was _______________ for a hot drink.

46. —Sorry, I didn‟t do well. I...

—You ________________________, and that‟s important.

47. —___________________________, Jack is the winner of the race. He looks really weak.

—You can‟t judge(判断)a book by its cover.

(二)根据中文意思完成句子。

48. 直到妇女儿童都登上救生艇,男人们才离开船。

The men ___________ the ship __________ all the women and children got into the lifeboats.

49. 他的英语说得和美国人一样好。

He _______________ English ________________ an American.

50. 当我在河边散步时,我看到有人正往河里乱丢垃圾。

When I _______________ along the river, I saw _______________ waste into it.

51. 他发现算出这道物理题不容易。

He __________________ it not easy ______________ this physics problem.

52. 我们一定要制止那些孩子在墙上乱涂乱画。

We must _______________________________________ on the wall.

六. 阅读与表达

When you go shopping, you get at least one plastic bag to carry what you bought. They are so common(普遍)that we don‟t often give them a second thought. Coles Bay, a small town in Tasmania, Australia, is trying to be the country‟s first plastic bag-free town. Since April 28, the small town‟s population(人口) Ben Kearney, a local businessman who supports(支持)the bag ban(禁令), said it would help people cut down on the use of plastic bags. “Most of the business here comes from the tourism and the good environment(环境)is important and necessary, so people are pretty supportive,”he said.

Every year Australians use more than 6 billion plastic bags. It takes years for the bags to biodegrade(生物降解). They kill about 100,000 seabirds and animals, which mistake the bags for food, every year.

Some Australian supermarkets have begun using fewer and fewer plastic bags in the past few months. Environmental groups are pushing for a plastic bag fee(收费)like that in Ireland. There, since plastic bags cost 10 pence(about 1 yuan)each, their use has been cut by 90 per cent.

阅读短文,回答下面的问题。

53. What kind of town is Coles Bay going to become?

______________________________________________________________________

54. How many people are living in the town?

______________________________________________________________________

55. Does the word“reusable”mean“that can be used again and again”?

______________________________________________________________________

56. Why does Ben Kearney support the bag ban?

______________________________________________________________________

57. After reading the passage, try to find another way to cut down on the use of plastic bags.

______________________________________________________________________

七. 书面表达

“五一”假期中,你作为一名志愿者(volunteer)来到公园里做好事,请记述你所做的事情。

根据题意,写出意思连贯,符合逻辑的短文。字数不少于50字。

所给英文提示词语供选用。

take pictures,

stop,

step on the grass, old people, pick flowers, handicapped(有障碍的,残疾的)

初二英语寒假作业(八)

一、单项选择

1. Mary is Australian, ____________ she lives in Germany.

A. but B. or C. and

2. —____________ do you write an English composition?

—Sometimes.

A. How long B. How often C. How much

3. —What‟s your favorite sport?

—Mountain climbing.

—Wow! It‟s exciting, but at the same time you ____________ not forget the danger.

A. can B. may C. must

4. —Would you like to go hiking with us?

—Sorry, I‟m very busy these days.

—____________. Go with us next time.

A. Oh, I‟m sorry, too

B. All right

C. It doesn‟t matter

5. —How is your English?

—Very poor. But I‟m ____________ hard ____________ it now.

A. studying; for B. working; on C. learning; from

6. —The weather by the sea never gets too hot during the summer, ____________?

—Yes, it‟s always cool.

A. doesn‟t it B. isn‟t it C. right

7. —____________ when you first met him?

—It was October 26th, 2004.

A. What day was it B. What time was it C. What was the date

8. —What are you going to the station for?

— My uncle is leaving for New York. And I want to ____________.

A. say goodbye to him

B. meet him

C. say hello to him

9. —Which subject do you like ____________, Chinese or maths?

—Of course maths. And I think it‟s the ____________ interesting of all subjects.

A. best; most B. better; most C. better; more

10. We won‟t go to the Great Wall if it ____________ tomorrow.

A. rain B. rains C. will rain

11. —What‟s ____________ on outside?

—We can see many old people in red and green dancing in the park.

A. going B. happening C. keeping

12. —The meeting starts at eight, ____________ it.

—No, it starts in half an hour.

A. does B. doesn‟t C. isn‟t

13. —Don‟t you think it interesting?

—What?

—The robot can ____________ Mr Mott do everything. And it will tell what it ___________ to his wife, Mrs Mott.

A. watch; sees B. ask; knows C. tell; finds

14. My father ____________ to America on business two days ago.

A. flies B. flew C. will fly

15. —When are you ____________ for your new house?

—Several weeks later, for we don‟t have to move in a hurry.

A. coming B. going C. leaving

二、阅读理解

(A)

Kangaroos live in Australia. They can‟t run and they can‟t walk. But they can jump more than nine metres a time. They can jump very fast—thirty kilometers per hour. Baby kangaroos are very small—only three centimeters long. They can‟t jump and they can‟t see. A young kangaroo is always in its mother‟s pouch for the first six months. They can see when they are nine weeks old. They can jump when they are eight months old.

Camels are perfect for desert. They have two rows of eyelashes to keep sand out of their eyes. They can close their long noses to keep out the sand, too. Camels have humps on their backs—some have one and some have two. Humps do not carry water. They carry food. If a camel is hungry, its hump will get smaller. If a camel is thirsty it will drink a lot of water at one time. A thirsty camel can drink up to one hundred and forty litres of water. If it drinks a lot of water, it won‟t drink again for over a week.

16. Kangaroos ____________ very fast.

A. walk B. run C. jump

17. A young kangaroo leaves its mother‟s pouch when it‟s ____________ old.

A. nine weeks B. eight months C. six months

18. Camels have ____________. So they are not afraid of strong winds in deserts.

A. eyelashes and long noses B. big bodies C. humps

19. There is ____________ in a camel‟s hump.

A. water B. food C. sand

20. A camel drinks ____________.

A. little water

B. water very often

C. a lot of water when it‟s thirsty

(B)

The weather is different in different parts of the world. In some places it is dry, and in others it is wet. If the weather is too dry, the land will not be good for animals or plants. In wet weather there may be too much rain. The rivers may go over their sides. The water may take the bridges away. If there is too much rain, and rivers go over their sides, a lot of people may not have enough food. If there is very dry weather for a long time, the riverbeds may be dry. If they are, there will be neither water nor fish for people. In some other parts of the world the weather may be very cold. There may be snow. If there is a lot of snow, the land and all the trees and buildings will be white. In the winter the days are very short and nights are long. On cold winter nights, when there are no clouds or winds, the sky is very clear. On those nights the moon and stars are very beautiful. People may put on their coats and go for long walks. When they come back to their houses, they may be happy to have hot tea and cake by their stoves(火炉).

21. In different parts of the world, people have ____________ weather.

A. dry B. wet C. quite different

22. When there is too much rain, a lot of people may not have enough food. Why?

A. Because the water in rivers may take the bridges away.

B. Because rivers go over their sides.

C. Because plants can‟t grow well in too much water in the fields.

23. Which of the following is not true?

A. People get little food in too wet or too dry weather.

B. People are afraid of too much water, but not a long time of dry weather.

C. Dry land is not good for plants or animals.

24. People go out for long walks when ____________ during the nights.

A. there are clouds in the sky

B. there are winds

C. there is moonlight

25. In the winter the sun rises ____________ in the summer.

A. later than B. earlier than C. at the same time as

(C)

Many years ago, people wanted to find a way by sea from Europe(欧洲)to China. In 1845, John Franklin left England with 134 men to look for a route(路线).

Franklin‟s ships had everything they needed. They had enough food for three years and thousands of bottles of lemon(柠檬)juice to stop disease. They also had two libraries with 3,000 books and good maps.

Franklin and his men left England on May 19th, 1845 and they sailed(航行)without any problems to Canada. When Franklin arrived at Baffin Bay(海湾) in July 1845, things were going very well. On July 26th, some sailors saw Franklin‟s ships when they were coming to the bay. That was the last time that anyone saw Franklin and his men alive(活着).

In England, many people wanted to know more about Franklin. Some people said that Franklin and his men were still living. Other people said that Franklin was certainly dead.

Jane Franklin was John Franklin‟s wife. She wanted to know what happened to her husband. In July 1857, she sent a group of people to look for her husband‟s ships. A year later, on an island in Baffin Bay, they found one of Franklin‟s sailors—ten years too late. He was lying in a boat near the sea—dead.

Not far away from the dead man, they found a small house. Inside it they found a note. It was the last message from Franklin and his men.

26. What kind of route did Franklin want to find?

A. A route by land from Europe to China.

B. A route by sea from Europe to China.

C. A route by sea from England to Canada.

27. What did they use lemon juice for?

A. Drinking. B. Medicine. C. Food.

28. What happened to Franklin and his men in Baffin Bay?

A. They ate up all the food in their ships.

B. Franklin and some of his men died of disease.

C. An accident happened to their ships.

29. People knew about Franklin‟s death ____________.

A. after 11 years of his death

B. in June, 1847

C. in July, 1857

30. When did Franklin‟s ships get to Baffin Bay?

A. On May 19th, 1845.

B. In July, 1845.

C. In September 1846.

三、完型填空

Mr. Brown and his wife had a small bar near a railway station. The bar didn‟t close ___1___ midnight because people came to drink while they were ___2___ for trains. So the business was good.

At three o‟clock one morning, a man was ___3___ sitting at the table in the bar. He was ___4___. Mr Brown‟s young wife wanted to go to ___5__. She looked ___6___ the bar several times, but the man kept sleeping. Then at last she went to her ___7___ and said to him, “You have tried to wake that man several times, and he isn‟t drinking ___8___. Why haven‟t you sent him away? It‟s too ___9___.”

“Oh, no, I don‟t want to sent him away,” he answered ___10___ a smile. “You know, each time I woke him up, he gave me five pounds. Then he went to sleep again.”

1. ( )

A. when B. until C. as D. while

2. ( )

A. looking B. asking C. waiting D. getting

3. ( )

A. only B. also C. still D. just

4. ( )

A. sleeping B. drinking C. talking D. thinking

5. ( )

A. home B. bed C. school D. work

6. ( )

A. for B. in C. around D. at

7. ( )

A. husband B. table C. room D. bed

8. ( )

A. too B. either C. anything D. everything

9. ( )

A. dark B. early C. cold D. late

10. ( )

A. in B. with C. for D. by

四、词形转换。

1. January is the ______________ month of the year. (one)

2. Please give ______________ an apple. (she)

3. They ______________ the film last night. (not see)

4. My car is ______________ than yours. (big)

5. Li Ming is ______________ boy in his class. (tall)

五、翻译句子。

1. 为什么不骑车去购物呢?

________________________________________________________

2. 好漂亮的公园啊!

________________________________________________________

3. 谢谢你帮我学数学。

________________________________________________________

4. 你最好不要告诉他这件事。

________________________________________________________

5. 我的电脑没出毛病了。

________________________________________________________

六、补全对话。

Mrs Jones: Hello.

Kate: 1. ____________________________ May I speak to Judy?

Mrs Jones: 2. ____________________________ Your call, Judy. It‟s from Kate.

Judy: Hi, Kate.

Kate: Hi, Judy. We‟re going to a dancing party tonight. 3. ____________________________

Judy: I‟d love to. 4. ____________________________

Kate: At the school gate at seven this evening.

Judy: OK. See you then.

Kate: 5. ____________________________

七、阅读与表达,阅读短文,并用完整的句子回答问题。

Hong Kong has about forty public beaches. Some of the beaches are among the best in the world. People can go there for a swim. You can go to most of them by bus. To go to some beaches you must take a boat. There are toilets, changing rooms and places to buy food and drink on most of the beaches.

You will swim there without danger if you remember these instruction:

1. Never swim alone.

2. Never swim after a meal or when you feel hungry or tired.

3. Do not stay in the water too long.

4. Never go out in a boat if you cannot swim.

Remember: A red flag means that it is dangerous for anybody to go into the water. A blue flag means that it is dangerous for children.

1. How many beaches are there in Hong Kong?

______________________________________________________________________

2. Can we reach all the public beaches by bus?

______________________________________________________________________

3. Will you swim after a meal?

______________________________________________________________________

4. What can‟t you do if you can‟t swim?

______________________________________________________________________

5. What does it mean when you see a blue flag?

______________________________________________________________________

八、书面表达

假设你叫李明,下周日是你的生日。你准备在家开一个生日晚会,并邀请部分同学和朋友参加。晚会7:30开始。你家住在牛街50号,乘626路或10路公共汽车在牛街下车。你家就在路的北边。房子是红色的,门是白色的,很容易找到。请根据所给信息,给你的好友林涛写一封不少于50个单词的信。

要点:

1. 邀请他参加晚会;

2. 晚会的时间和地点;

3. 你的地址;

4. 如何到达;

5. 你很希望他来。

注意:信的内容必须包括以上要点,语意连贯,句式规范,信件格式正确。

初二英语寒假作业(九)

一. 单项选择

( )1. We enjoyed at the party yesterday.

A. we B. us C. our D. ourselves

( )2. She can help me my homework.

A. to B. on C. with D. for

( )3. —Coffee is ready. How nice it ! Would you like some?

—Yes, please.

A. feels B. smells C. sounds D. looks

( )4. How about to the radio or reading a newspaper in English?

A. listening B. listen C. to listen D. listened

( )5. There are some people in the park, ?

A. are there B. aren‟t there C. is there D. isn‟t there

( )6. We should always English in class.

A. speak B. to speak C. speaking D. spoke

( )7. They couldn‟t wait their new classmate.

A. meet B. meets C. meeting D. to meet

( )8. Our English teacher to England. She‟ll come back in ten months.

A. have been B. has been C. has gone D. have gone

( )9. It‟s a good idea your vocabulary notebook every day.

A. check B. to check C. checking D. checked

( )10. The teacher a gift, but he didn‟t it.

A. accepted; accept B. accepted; receive

C. received; receive D. received; accept

( )11. —I called you yesterday evening, but there was no answer.

—Oh, I am sorry, I dinner in my friend‟s home.

A. have B. had C. was having D. have had

( )12. —What are you going to do in Spring Festival?

—I am not sure. I make a trip to Tianjin with my friends.

A. will B. possible C. might D. must

( )13. —Could you give me a hand, please?

—Sure. What would you like me ?

A. do B. to do C. doing D. does

( )14. The film has for about twenty minutes.

A. started B. begun C. been D. been on

( )15. — ?

—It‟s snowy.

A. What‟s the weather like B. What does she look like

C. What‟s the matter D. How is it about

二. 完型填空

Everyone in our school loves sports. Every morning we get up, we do morning exercises. After the second class we do exercises again. We only have classes twice a week, but we do physical training (锻炼) at five every afternoon. The most popular sport is basketball. The enjoy playing it and many of the girls like it, too. popular sport is football and in every class there a lot of football fans (球迷). volleyball is played when the weather is fine. We have school teams in

basketball, football and volleyball. Our teams often friendship (友谊) matches with the teams from other schools. When there is a match, of us go to watch it and cheer (喝彩) our side on.

Beside (除„„之外) ball games, some of us like track-and-field events (田径项目), we often practice running, jumping and throwing. Every term we have tests in these events and once a year we hold a sports meeting.

Sports help us to keep .

( )1. A. while B. when C. before D. after

( )2. A. English B. Chinese C. P.E. D. music

( )3. A. teacher B. boys C. students D. people

( )4. A. Some other B. Other C. Another D. The other

( )5. A. are B. is C. were D. was

( )6. A. Never B. Often C. Usually D. Always

( )7. A. see B. have C. look at D. watch

( )8. A. several B. few C. a little D. many

( )9. A. because B. but C. or D. and

( )10. A. thirsty and hungry B. healthy and happy

C. fat and tired D. thin and weak

三. 阅读理解

A

We each have a memory. That‟s why we can still remember things after a long time. Some people have very good memories and they can easily learn many things by heart, but some people can only remember things when they say or do again and again. Many of the great men of the world have got surprising memories.

A good memory is a great help in learning languages. Everybody learns his mother language when he is a small child. He hears the sounds, remembers them and then he learns to speak. Some children are living with their parents in foreign countries. They can learn two languages as easily as one because they hear, remember and speak two languages every day. In school it is not so easy to learn a foreign language because the pupils have so little time for it, and they are busy with other subjects, too.

But your memory will become better and better when you do more and more exercises.

( )1. Some people can easily learn many things by heart because .

A. they always sleep well B. they often eat good food

C. then read a lot of books D. they have very good memories

( )2. Everybody learns his mother language .

A. at the age of six B. when he is a small child

C. after he goes to school D. when he can read and write

( )3. Before a child can speak, he must .

A. read and write B. make sentences

C. hear and remember the sounds D. think hard

( )4. In school the pupils can‟t learn a foreign language easily because .

A. they have no good memories B. they have no good teachers

C. they don‟t like it D. they are busy with other subjects

( )5. Your memory will become better and better .

A. if you have a lot of good food

B. if you do more and more exercises

C. if you do morning exercises every day

D. if you get up early

B

The following are three ads (广告):

Driver Wanted

1. Clean driving license

2. Good-looking

3. Age over 25

Apply (申请)to: Capes Taxi, Shenzhen Tel: 0755-6561382

Air Hostess (空姐) Wanted

1. Age between 20 and 33

2. Height from 1.6m to 1.75m

3. Two foreign languages

4. College graduate

Apply to: China Airlines, Beijing

Tel: 010-88488970 Teacher Needed For private language school Teaching experience necessary Apply to: Instant Languages Ltd, Dalian Tel: 0411-4313861

( )6. If you want to work in the south, you can apply for a job as .

A. a driver B. an air hostess C. a teacher D. all of the above

( )7. You may call when you wish to be a teacher.

A. 0755-6561382 B. 0411-4313861

C. 010-88488970 D. A and B

( )8. Mary, aged 26, knows English and Japanese, which job can be given to her?

A. Driving for China taxi. B. Working for China Airlines.

C. Teaching at instant languages Ltd D. None of the above.

( )9. What prevents Jack, an experienced taxi driver, working for Capes Taxi?

A. Liking beer and wine.

B. Breaking traffic rules.

C. Being unable to speak a foreign language.

D. Not having college education.

( )10. Which of the following is not mentioned in the three ads?

A. Height B. Age C. Language D. Health

C

Happiness is for everyone. You don‟t need to care about those people who have beautiful houses with large gardens and swimming pools or those who have nice cars and a lot of money and so on. Why? Because those who have big houses may often feel lonely and those who have cars may want to walk on the country roads at their free time.

In fact, happiness is always around you if you put your heart into it. When you are in trouble at school, your friends will help you; when you study hard at your lessons, your parents are always taking good care of your life and your health; when you get success, your friends will say congratulations to you; when you do something wrong, people around you will help you to correct it. And when you do something good to others, you will feel happy, too. All these are your happiness. If you notice a bit of them, you can see that happiness is always around you.

Happiness is not the same as money. It is a feeling of your heart. When you are poor, you can also say you are very happy, because you have something else that can‟t be bought with money. When you meet with difficulties, you can say loudly you are very happy, because you have more chances to challenge (向„„挑战) yourself. So you can not always say you are poor and you have bad luck. As the saying goes, life is like a revolving (旋转) door. When it closes, it also opens. If you take every chance you get, you can be a happy and lucky person.

( )11. Those who have big houses may often feel .

A. happy B. lonely C. free D. excited

( )12. When you fall down in a PE class, both your teacher and your classmates will .

A. laugh at you B. play jokes on you

C. quarrel with you D. help you up

( )13. What will your friends say to you when you make great progress?

A. Oh, so do I. B. Congratulations.

C. Good luck. D. It‟s just so-so.

( )14. Which idea is NOT right according to the passage?

A. People who have cars would never like to walk in the open air.

B. You can get help from others when you make mistakes.

C. You can still be a happy person even if you have little money.

D. Happiness is always around you though difficulties come towards you.

( )15. Which of the following is this passage about?

A. Bad luck. B. Good luck. C. Happiness. D. Life.

. 四、从括号中选择恰当的词填空

1. I‟m very thirsty. Please 2. Don‟t go (through, across) the road when the red lights are on.

3. I was very 4. These 5. My father made me (go, to go) to bed early.

五. 完成句子

1. 当你到这个国家学一门外语就很容易。

2. 学习英语用了我很长时间。

3. 你能给我看看那份地图吗?

Could you ?

4. 这件毛衣摸上去很柔软。

This sweater .

5. 你想要听乡村音乐吗?

六. 看图完成对话

W: Excuse me, where is the nearest supermarket, please?

M1: Sorry, I don‟t know. W: Thank you all the same. Excuse me, ?

M2: Go down this street. . Then you‟ll see the super market. It‟s between And you can go there by bus.

W: ?

M2: Over there on the other side of the street.

W: I see. Thank you very much.

七. 阅读与表达

I was on my way to the Taiyetos Mountains. The sun was going down when my car broke down near a faraway village. I was thinking about my bad luck and didn‟t know where I was going to spend the night when the villagers came up to me, asking me to go to the houses of their own as if I were their best guest (客人). At last I went into an old woman‟s home. She lived alone in a little house. While I was having a rest, the villagers took my car with their cows to a nearby car repair shop.

I had noticed (注意到) there were three chickens walking in the garden that evening but only two were left alive (活着的) later. One of them was on my dinner table in the evening. Other villagers brought me a lot of food to eat. Though I didn‟t know their language, we drank happily together till far into the night. The following day the villagers brought back my car.

When the time came for me to say goodbye to my friends in the village, I wanted to give some money to the kind old woman, as I gave her so much trouble. She got angry and said she couldn‟t take the money.

根据短文内容,回答下列问题。

1. Where was the writer traveling?

2. When was his car broken?

3. Where had one of the old woman‟s chickens gone?

4. Why couldn‟t they talk to each other?

5. Why didn‟t the old woman take the money?

八. 书面表达

校刊要求每位同学写一篇短文介绍自己最要好的朋友,请以“My best friend, Mike”为题写好介绍,词数不少于60个。短文应包括下列所有要点提示:

1. 他的外貌特征:高而瘦,圆脸,大眼睛,小鼻子,脸上总带着笑容„„ 2. 他的个性特点:诚实,保守秘密,乐于助人,而且幽默„„ 3. 他的理想:长大想当一名运动员„„

初二英语寒假作业(十)

一. 选择填空

1. I‟d like to go shopping with you, I‟m too busy today. A. and B. but C. or D. so 2. Welcome to our hotel. It‟s one in the city. A. good C. better C. best D. the best 3. The children will climb the hill if it tomorrow. A. won‟t rain B. didn‟t rain C. doesn‟t rain D. isn‟t raining 4. They can go to school A. under B. with C. in D. to 5. —What good weather! Let‟s go boating on the lake. — A. That‟s all right B. Thank you very much C. It doesn‟t matter D. That‟s a good idea 6. —What‟s your brother doing in his room now? —He A. makes B. made C. is making D. will make 7. A. How many B. How often C. How much D. How far

8. It five years since I saw him last time. A. was B. are C. is D. were 9. I‟m late, but I on time tomorrow. A. came B. come C. will come D. have come 10. She has got a cold but she will get well A. after B. before C. in D. for 11. —How about your trip to the seaside? —Great! I enjoyed A. swimming B. to swim C. swim D. swims 12. —We can go to Beijing Railway Station by bus. —Why not A. walk B. walking C. to walk D. walked 13. —He didn‟t come to the meeting, he? — A. didn‟t; Yes B. did, Yes C. didn‟t; No D. did; No 14. The student Jane is from England. A. naming B. to name C. names D. named 15. —I haven‟t seen you for quite a long time. —I am busy A. write B. writing C. written D. to write 16. The students are very tired. Let them a rest. A. to have B. have C. having D. had 17. Mr. Yu often has sports on Sunday morning, A. does B. doesn‟t C. has D. hasn‟t 18. —you —Yes. I A. Have, returned; returned B. Do, return; returned C. Will, return; have returned D. Did, return; returned 19. My family will go to Beidaihe th. A. in B. at C. on D. by 20. We a meeting at eight yesterday. A. are having B. will have C. were having D. had

21. After having the soup I cooked, mum said, “Wow, it A. tastes B. seems C. smells D. feels 22. —You have caught a cold. Is it very serious? —No, A. nothing serious B. serious nothing C. anything serious D. serious anything 23. The sweater feels very . A. soft and comfortable B. softly and comfortably C. softly and comfortable D. soft and comfortably 24. The old man told the boys football on the street. A. not play B. not to play C. don‟t play D. didn‟t play 25. What will the weather A. like B. be likely C. be D. be like

26. I took Mr. Black to the hospital this morning, A. because B. and C. why D. so 27. —are you going in Beijing?

—We are going to the Summer Palace and the Great Wall. A. Where B. What C. Why D. When 28. Could you help A. I, my B. me, me C. me, my D. my, I 29. The good news made us very . A. excited B. exciting C. to excite D. be excited 30. —Must I finish that work today?

—No, you A. mustn‟t B. needn‟t C. shouldn‟t D. couldn‟t

二. 完型填空。

A

Mary was quite busy yesterday. She up before 7:00 in the morning. She her face very quickly and had some milk and bread for . It was a fine day. She went to early. She had four classes in the morning, maths, Chinese, English and PE. She didn‟t go back home for lunch. She just had it at school. She had an easy lunch. Then she had a little rest, but she worked very hard in class . School was over at half past four. She played basketball after school with her classmates and then went home. home she bought a new golden pen. When she home she had a short rest. Because her mother was not at home, she had to cook supper quickly and supper. Then she did all her homework and went to before ten. 1. A. get .B. gets C. got D. getting 2. A. washed B. cleaned C. washes D. cleans 3. A. lunch B. supper C. breakfast D. dinner 4. A. cinema B. office C. school D. bank 5. A. all day B. the all day C. all the day D. all a day 6. A. To her way B. By her way C. On her way D. On way 7. A. got B. reached to C. reached D. gets 8. A. did some cleaning B. doing some cleaning C. clean a house D. to clean the house 9. A. watched B. read C. saw D. looked 10. A. bed B. beds C. the bed D. a bed

B

Cars are very popular in America. When the kids are fourteen years old, they dream of having their own work after school to a car. In most places people learn to drive in high school. They have to take a test to get a license. Learning to drive and getting a driver‟s may be one of the most exciting things in their lives. For many, that piece of paper is an important symbol that they are now grown-ups.

Americans seem to love their cars almost more than anything else. People almost never go to see a doctor when they are But they will take cars to a “hospital” at the smallest sign of a problem. At weekends, people most of the time in washing and waxing (打蜡) their cars. For some families it is not enough to have car. They often have two or even three. Husbands need a car to go to work. Housewives need a car to go shopping or to take the children to school or 1. A. cars B. computers C. bikes D. houses 2. A. borrow B. buy C. lend D. sell 3. A. old B. tall C. strong D. young 4. A. language B. listening C. driving D. body 5. A. address B. book C. name D. license 6. A. sick B. healthy C. pleased D. angry 7. A. his B. her C. your D. their 8. A. cost B. take C. spend D. pay 9. A. no B. one C. some D. several 10. A. other B. another C. others D. else

三. 阅读理解

A

It was 10:30 at night. Mrs. White put down her book and went into her 14-year-old son‟s room. Tom was sitting in front of a bright computer screen on which a colourful dinosaur jumped and shouted.

“Oh, Tom! You‟re still playing. You must stop and go to bed now. If not, you‟ll be very tired tomorrow.” Said Mrs. White. “But I‟ve nearly beaten the dinosaur.” Said Tom.

“You‟re always playing on that computer. You spend more time with this machine than with us,” she said with a smile. “What‟s so special? Show me what it can do!”

Tom was very excited. “I think it‟s a great computer, Mum!” he said happily, “The hardware is good. There‟s so much memory that I can put in a lot of software programs. This game, DINOSAURSPLAYER, is my favorite. It‟s about a super boy exploring the moon with the help of his knowledge in maths and physics. It‟s interesting and helpful. Also, I can download lots of information from the Internet so that I can know what is happening at home and abroad. Now let me show you!” Tom began pressing keys on the keyboard. The screen changed in answer to his orders.

“Oh, Tom,” laughed Mrs. White. “I‟m sure it‟s a wonderful computer, but I‟m afraid I don‟t know much of what you‟re talking about.” 1. When Mrs. White entered Tom‟s room, he was

A. lying in bed

B. standing in front of the computer C. playing computer games D. reading a book

2. From the passage we know that the story happened A. before Tom went to school B. before Tom went to bed C. just after Tom had supper D. after Tom slept for hours

3. Tom was very excited because A. he beat the dinosaur

B. he was worried about his computer game

C. his mother began to feel interested in the computer D. his mother asked him to go to bed 4. What will be the possible end of the story? A. Tom will never play computer games again.

B. Mrs. White will never ask Tom to go to bed early. C. Tom will not get on well with his mother.

D. Mrs. White and Tom will understand each other better.

B

Over 1,400 university students from 37 countries and regions came to China for the 2006‟s WorldMUN meeting (世界大学生模拟联合国大会). The meeting was held in Peking University from March 27th to 31st, 2006.

The idea of the WorldMUN began in 1991. A group of students at Harvard University wanted to make a new meeting to bring together the students who were interested in the world. They wanted to discuss different problems around the world. The WorldMUN is based on Last year, it was held in Scotland.

This was the first time the WorldMUN meeting was held in Asia. It was also the biggest one in history. The students came from the best universities around the world. During the five-day meeting, they did everything that people in the UN do. Though their skin colors and cultures were different, they had the same wish to make the world better. 5. The 2006‟s WorldMUN meeting was held in A. Scotland B. Harvard University C. Poland D. Peking University

6. The word “compromise” in the passage means in Chinese. A. 友谊 B. 理解 C. 和解 D. 屈服 7. Which of the following is NOT true?

A. The students want to travel around the world. B. The students are interested in the world.

C. The students want to discuss different problems. D. The students wish to make the world better. 8. The best title of the passage is . A. The First WorldMUN Meeting B. The WorldMUN Meeting C. University Students

D. Problems Around the World

C

The modern summer Olympic Games which was held in Barcelona, Spain in 1992, was the 25th since it was first held in Athens, Greece in 1896. Do you know the dates and the places of all the 25 Games, and also the total of gold medals (金牌总数) the first three

*Note: GDR is East Germany.

9. By the year 1992, the modern summer Olympic Games had been held times in USA. A. one B. two C. three D. four

10. How many gold medals did East Germany win in the summer Olympic Games? A. 124. B. 144. C. 198. D. 200. 11. From the data (资料) we can see that .

A. every country should hold the Olympic Games in turn

B. any Olympic Games should not be called off (取消) for any reason C. no peace, no sports

D. if there is a war anywhere, the Olympic Games must be stopped

D

Maybe you know floods, droughts, earthquakes, sandstorms and so on. But have you ever heard of typhoons?

Typhoons are some of the worst storms, usually around the Pacific Ocean and the South China Sea, most happen in July, August and September.

How does a typhoon happen? When lots of sea water gets hot in the summer sun, it evaporates into the air. This makes the air hotter. After the air gets warmer, it starts to move quickly, making wind. The wind goes in circles, and it keeps moving higher in the sky. The warmer the air gets, the quicker the wind moves. And when the wind moves faster than 30 meters a second, a typhoon begins.

A typhoon has two parts. One is called the “eye”. In the eye, the wind does not move so fast. The other part is the wall of clouds around the eye. This is where the strongest and heaviest rains are.

Typhoons are very dangerous. In 2004, Typhoon Yunna killed 164 people in Zhejiang, and 24 people were missing. In May 2006, Typhoon Pearl hit Guangdong and Fujian, and thousands of houses were damaged.

What to do when a typhoon hits?

Stay inside, Close all the windows and stay away from them.

Try to bring all of your things inside. Strong winds could even blow away your bikes! Listen to the radio or TV for important information. If you‟re told to go to a safer place, do so right away. 12. Typhoons often happen . A. in the hot season B. in the cold season C. only in China D. under the sea

13. Which picture can tell meaning of the underlined sentences “…, it starts to move higher up into the sky. Then, cooler air around it rushes in.”?

14. When a typhoon comes, we cannot A. close all the windows

B. listen to typhoon news

C. stay outside with our bikes

D. stay inside the strong house

15. Which of the following is TRUE according to the passage?

A. The warmer the air gets, the slower the wind moves.

B. This year Typhoon Peal killed 164 people in Zhejiang.

C. The strongest winds and heaviest rains are in the eye of typhoon.

D. A typhoon will happen when the wind goes faster than 30 meters a second.

四. 根据句意,用所给单词的适当形式填空。

1. Mr. Smith gave us some (建议) on how to learn English.

2. Liu Shanzi can‟t get an 教育) because his family is poor.

3. Pandas live in the forests and mountains of (西南) China.

4. It is 可能) that we can see the film this weekend.

5. We learn a lot at the 博物馆).

6. Pandas mainly live on . (bamboo)

7. The fishermen are told to be more careful on days. (wind)

8. Of all the subjects, which do you think is (difficult) to learn?

9. Yan Liwei returned to the earth (safe).

10. Please make a (shop) list before you go out to buy things.

五. 动词的适当形式填空

1. I don‟t need 2. —When —The day after tomorrow.

3. —Who (knock) at the door?

—It‟s me.

4. He (use) his bike for twenty years, but he doesn‟t want to buy a new one.

5. —When the plane (leave), sir?

—I‟m not sure. Maybe five minutes ago.

6. In many parts of China, it often (rain) a lot in summer.

7. When I visited them last night, they 8. The teacher was busy 9. —Where is Mr. White? I haven‟t seen him for a few days.

—He 10. Would you please

六. 句型转换 (对画线部分提问)

is the Yellow River?

2. The old man knew that too much salt was harmful (改为否定句)

The old man that too much salt was harmful.

3. This dress cost her 90 yuan. (改为一般疑问句)

4. Tom has cleaned the room, ? (反意疑问句) (划线部分提问)

the little boys play football yesterday?

七. 翻译句子

1. 无论刮风下雨,Tom总是第一个到校。

Whether it is windy or rainy, Tom is always the first to school.

2. 班会已经开了半个小时。

The class meeting has for half an hour.

3. 春天树叶变绿了。

The leaves in spring.

4. 我能把你做的空间站模型拿给我父母看吗?

your space station to my parents?

5. 夏威夷因美丽的海滩而闻名。

Hawaii is its beautiful beaches.

八. 根据语境,用适当的语言完成对话。

A: It‟s a fine day today. Let‟s go boating.

B: Sorry, .

A: All right. But which shop are you going to?

B: Mr. Wang‟s shop.

A: But it‟s very far and we have only one bike.

B: Let‟s put it away and A: Oh, it‟s half past two. Let‟s hurry. B: Why?

A: There is going to be a football match at half past four. .

B: All right. Let‟s go at once.

九. 阅读回答问题。

28 High Road

Oxford

Sept. 20, 2006

Dear John,

Thank you very much for your letter. I‟m glad you enjoyed your holiday with me.

We enjoyed having you and your sister here. We hope that you will both come again next year. Perhaps you can stay longer next time you come. A week is not really long enough, is it?

I hope you are both working hard now that you‟re in Grade Three. I shall have to work hard next year when I am in Grade Three. Tom and Ann will be in Grade Three the year after next.

They went on a picnic yesterday but I didn‟t go with them because I hurt my foot and I couldn‟t walk very well. They went to an island and enjoyed themselves there. Do you still remember the island? That‟s where all five of us spent our holiday.

Tom, Ann and I send our best wishes to Betty and you. We hope to see you soon.

Yours sincerely,

Peter

根据短文回答问题,用整句。

1. How long did their holiday last this year?

2. Who stayed with Peter for the holiday?

3. When did Peter write the letter?

4. Who is in Grade Two now?

5. Why didn‟t Peter go to the island for picnic?

十. 写作

请你参考所给信息,写一篇关于你的同学或朋友的小短文。

要求:

1. 短文中不得出现你的真实姓名。

2. 必用所给信息,可适当发挥。

3. 词数:60~80词。

信息:Zhang Li: 14; tall/ short; thin/ strong; kind; favorite sport; listen to music; read books…

初二英语寒假作业(一)

一. 词汇.

A.根据所给中文写出正确的单词。

1. We got off the ____________________(长途汽车)and went into the park.

2.What do you think is the most ___________________(不重要的) in poor countries ?

3.I hope you can give me some ______________________(建议).

4.When he is hungry ,he always_____________________(敲)on the cage.

5.They are all the ___________________(英雄) in the history.

7. We must remember those ____________________.

8. You can have some ____________________lessons at the school.

9. He doesn‟t know how to look after____________________ .

10. Do you think Betty is __________________ of her three sisters?

12._________________________in a big city is very interesting.

13. I________________________ an interesting film at present.

14. He never __________________ out at night.

15. David________________________ off the bus yesterday.

二. 选择题

( ) 1. ---Mum, may I watch TV now?

---No, you finish your homework first.

A. mustn‟t B .must C. need D. can‟t

( ) 2. ---You look today, Julia!

---Yeah, I‟ve got an MP3.It‟s a birthday present from my parents.

A. sad B. worried C. happy D. tried

( ) 3. ---Why did you come to school late this morning?

I watched the Football World Cup until 12 o‟clock last night.

A. If B. Because C. Since D. though

( ) 4. It is ____useful book. 1 borrowed it from ____school library. A. an; a B. a; an C. a; the D. an; the

( ) 5. We can do the work better with____ money and ____ people.

A. less, fewer B. fewer, less C. less, less D. fewer, fewer

( ) 6. While I____ with my friend, she came in.

A. am talking B. was talking C. talked D. am going to talk

( ) 7. All of us enjoy playing computer games, ___ we can't spend too much time on it.

A. and B. or C. but D. so

( ) 8. Would you please tell me____?

A. what was her name B. what her name was

C. what is her name D. what her name is

( ) 9.— We can use QQ to talk with each other on the Internet.

— Really? Will you please show me ____ it?

A. what to use B. how to use C. how can I use D. what can I use

( ) 10.—Mum, could I have an MP3 like this?

—Certainly, we can buy one, but as good as this. The price of this kind is a little high.

A. a cheap B. a cheaper C. a small D. a smaller

( ) 11.—Can I help you? —I‟d like A. two pair of shoes B. two pairs of shoe

C. two pair of shoe D. two pairs of shoes

( ) 12.—You watched the fashion show last night, didn‟t you?

— I missed the beginning.

A. so B. and C. but D. then

( ) 13.I feel it hard to keep up with my classmates. But whenever I want to , my teacher always encourages me to work

harder.

A. go on B. give up C. run away D. give back

( ) 14.How time flies! Three years A. is B. are C. was D. were

( ) 15.---Mum, we‟re going to visit an island this weekend. --- !

A. Goodbye B. That‟s OK C. Not at all D. Have a good trip

三. 完型填空

You really have to get very old before you find you're old. I'm in my middle fifties and I don't feel old yet. However, sometimes I look

2. A. today's

3. A. gone

4. A. made

5. A. making

6. A. untrue

7. A. movie

8. A. lost

9. A. As usual

10. A. movies

四. 阅读理解 B. yesterday's B. new B. decided B. watching B. loud B. television B. increased B. Above all B. food C. tomorrow's C. expensive C. hoped C. preparing C. high C. radio C. watched C. Besides C. cars D. future's D. popular D. tried D. buying D. realistic D. telephone D. played D. Especially D. televisions

A

When Kyle walked into Ernie‟s Pet World, he looked very anxious. The shop owner, Ernie, jumped out of his seat to greet Kyle. He was the shop‟s first customer of the day.

“Good morning, sir!” Ernie said, “What can I help you with?”

“Well, I ……”Kyle started to say.

“Wait, don‟t tell me,” Ernie stopped him. “You‟re looking for a …a little dog… for your daughter‟s birthday, Right?”

“Not really, I just….”

Ernie didn‟t let him finish. “Ah, I‟ve got it, You just moved to a new office, and you want some fish for it, I have some very nice fish bowls over here.”

“In fact, I…”Kyle was starting to look very nervous and strange.

“No fish? Ah, a cat! You look like a cat person. At Ernie‟s Pet World, we have the best cats. Take a look at this lovely one with long, white hair, She‟s looking at you. She‟s thinking, „Take me home. Take me home.‟ Would you like some cat food and toys, too?”

“No, thank you,” Kyle said. By this time, he was walking up and down. He had a terrible look on his face. “Really, I‟m not interested in cats or fish or little dogs.”

“What do you want, then?” Ernie asked.

Kyle looked like he was going to cry. “I just want to know if can use your toilet!” he said at last.

( )1. Which is TRUE about Ernie‟s Pet World?

A. It has many workers. B. Kyle goes there often.

C. It sells school things. D. Ernie is the owner of the shop.

( )2. What kind of person is Ernie?

A. Very lazy. B. Rather shy. C. Over warm-hearted. D. Not friendly.

( )3. Kyle went into the Ernie‟s Pet World to .

A. buy a little dog B. take fish home

C. meet the shop owner D. look for a washroom

( )4. What does the underlined word “anxious” mean in Chinese?

A.焦急的 B.兴奋的 C.生气的 D.失望的

( )5. What is the best title of the story?

A. Kyle and Ernie B. Pet or Toilet

C.A Pet Lover D.A Busy Pet Shop

B

Do you know Fei Junlong and Nie Haisheng? Fei Junlong was born in 1965. He is a Chinese astronaut selected(选拔)as a member of the Shenzhou program. He was born in Jiangsu and joined the People‟s Liberation Army Air Force(中国人民解放军空军)in 1982 at the age of 17. And since then he has only returned home six times. As a fighter pilot(战机飞行员), he was selected to an astronaut in 1998.

He married Wang Jie in 1991 and has one son.

Nie Haisheng was born in September 1964. He is also a Chinese astronaut selected for the Shenzhou program. He is from Hubei. After graduating from high school he joined the People‟s Liberation Army Air Force, and became a fighter pilot. In 1998 he was selected for the astronaut program.

He was one of the three astronauts who were in the final group to train for the Shenzhou 5 fight. Yang Liwei was also picked out for the flight, with Zhai Ahigang.

On September 23,2005 it was reported that Zhai and Nie would be one of the three pairs of astronauts who would be in the final training for Shenzhou 6. But in the end, Fei and Nie were chosen, and Zhai missed his chance to space again.

Nie Haisheng got married to Nie Jielin and has an 11-year-old daughter.

( ) 6. When did Nie join the army?

A. In 1982. B. In 1998.

C. In 1991. D. After his graduation from high school.

( ) 7. Who among the astronauts in this passage hasn‟t been to space?

A.Zhai Zhigang. B.Nie Haisheng. C.Yang Liwei. D.Fei Junlong.

( ) 8. How many times has Fei Junlong returned home since he left at the age of 17?

A. Three times. B. Four times. C. Six times. D. Eight times.

( ) 9.How many people did the writer mention in this passage?

A.5. B.6. C.7. D.8.

( ) 10.Which of the following is TRUE according to this passage?

A. Yang Liwei was born in Jiangsu.

B. Nie Haisheng has an elever-year-old daughter.

C. Fei Junlong is one year older than Nie Haisheng.

D. Zhai Zhigang got married to Nie Jielin and has a son.

阅读下面短文,根据短文内容回答下列各题。

11.What telephone number should you call if you need a 2-bedroom flat?

_______________________________________________________________________________

12.Mr. Chen lives in Hong Kong. He has never traveled abroad before and he wants to make a tour around Europe. Which travel company can help him?

_______________________________________________________________________________

13.How much does it cost to make a seven-day tour around five countries in Europe?

_______________________________________________________________________________

14.If Mr. Bell wants to be a business journalist for World watch, what languages does he have to know well?

_______________________________________________________________________________

15.If Miss Jones has worked as a journalist for one year, can she be employed(聘用)by World watch?

_______________________________________________________________________________

五. 作文

请以 “My life school”为题写一篇80词左右的文章。

提示词:get up , have breakfast , go to school , have lessons ,

practice…doing , do…activity ,do…exercise ,

chat…with ,listen/watch , enjoy ,

初二英语寒假作业(二)

一、词形变化 根据句意﹑汉语注释及所给单词,在空格内写出各单词的正确形式。

1. Tim and I are good friends, and we keep our secrets to _________ (we).

2. The girl lost her money on her way home yesterday. How _______ she was! (care)?

3. My cat weighs two kilos. But I don‟t know my son‟s ___________. (weigh)

4. Tom has the _________ milk of us three. (little)

5. Amy is becoming much _________after playing softball. (health)

6. ________ (幸运的是), he was not hurt badly.

7. They got the first place, so they were the __________(获胜者)of the football final.

8. We will remember the _________ (令人愉快的)trip forever because we really had a good time during the trip.

9. Jim is ill. He has to take some ________( 药 )three times a day.

10. The 广告 )in this newspaper are more interesting than those in that one.

二、单项选择 在A、B、C、D四个选项中,选出可以填入空白处的最佳选项。

( )1. John is ______ honest boy. He has ______useful book.

A. an ; an B .a ;a C .a ;an D .an ;a

( )2. Most of the workers like driving or riding to work,______ I like walking.

A. and

A. Let‟s go B. so C. but D. or B. I‟d like to

D. It‟s a pleasure

B. What; worse ( )3.---Why not join us in the game, Kitty ?---________, but I have to do my homework first. C. Yes, please A. What a ; bad ( )4. --________ terrible weather it is! --The radio says that it‟ll get _________later.

C. How; bad D. How a ; worse

( )5. Is there _______ about animals on the Internet?

A. special something B. something special

C. special anything D. anything special

( )6. Don‟t speak to him like that , he‟s only an___________ boy.

A. 8 years old B. five-year-old

C. eleven-year-old D. eight-years-old

( )7. The number of the students in Woodland School__________ than before.

A. are more B. are larger C. is fewer D. is smaller

( )8. I don‟t want the bananas. They smell _____.

A. well B. good C. terrible D. badly

( )9. Do you think some special dogs can do things __________ man?

A. as well as B. as good as C. as soon as D. as careful as

( )10. What did you do in the garden? I watched John ______ his bike.

A. to mend B. mended C. mend D. mends

( )11. The trip from the school _____ about half an hour by coach. It was too ______.

A. spends; bored B. takes; bored C. spent ; boring D. took; boring

( )12. ----How did Daniel 1earn how to make a homepage? ----Oh,.

A.taught himself B.learnt himself C.learn by himself D.taught by himself

( )13. He looks so tired ______ too much computer work.

A. because his B. because of C. because it‟s D. because for

( )14. ----Which of the two boxes is heavier? ----This one is heavier than A. another B. the other C. others D. other

( )15. He ________ his homework today.

A. need to finish

三、动词填空 用括号内所给动词的适当形式填空。

1. The 2008 Olympic games_____(take) place in Beijing ,all the Chinese are looking forward to it.

2. The Greens will fly back to their home tomorrow, so they are busy ______(pack) their bags.

3. It is important for wild animals __________ (build) more reserves (自然保护区).

4. Keep ______ (work) on the problem, you will solve it soon. B. needs finishing C. needn‟t to finish D. doesn‟t need to finish

5. Last Sunday, Mum made me ________ (do) some washing at home.

6. It‟s 4:30 p.m. The students of Class 4 _______(play) football on the playground.

7. Mother asks him ______ (not forget) to lock the door when he leaves home.

8. There‟re so many wonderful clothes in the shop, so she can‟t decide which one ____________(choose).

9. They wanted to run away but we _______(catch) them in the end.

10. Linda‟s mother will come back if she ________ (finish) her work this week.

四、完成句子 按所给的汉语,用英语完成下列句子,1-4题每个空格只能填一个单词。

1. 我们应该鼓励孩子们保护动物。

We should__________ children __________ __________ animals.

2. 我愿意和我的朋友分享快乐。

I‟m ______ _______ ________my joy ________ my friend.

3. 我决定呆在家里上网,不出去购物了。

I _______to stay at home and search the Internet _______ _____ _______ shopping.

4. 在下了车之后,他们不再感到恶心了。

After they _______ ________ the coach, they didn‟t feel ______ _______ ________.

5. 西蒙的衬衫和迪克的衬衫颜色一样。

Simon‟s shirt ________________________________________.

6. 你到那儿后,请尽快给我打电话。

After you arrive there, please call me____________________________.

五、完形填空

A teacher was asking a student a lot of questions,.The teacher then decided to ask him some .

“ she asked.

The student for some time and then answered, “An important Person.”

“No.it is a battle(战役).”the teacher said.,it.Then she asked,“Who was the first president of the United States?” The student thought and thought ,but didn‟t.The teacher got very angry and shouted,“George Washington!”

“Come back!” the teacher said.“didn‟t tell you to go.” “Oh,I‟m sorry,” the student said , “I thought you the next student.”

( )1. A.wouldn‟t B.couldn‟t C.shouldn‟t D.mustn‟t

( )2. A.1ittle B.few C.a little D.a few

( )3. A.What B.Which C.Who D.Where

( )4. A.thought B.spent C.waited D.stopped

( )5. A.angry B.worried C.sad D.happy

( )6. A.find B.1ose C.do D.show

( )7. A.speak B.tell C.say D.talk

( )8. A.friend B.seat C.address D.side

( )9. A.He B.She C.I D.We

( )10. A.called B.taught C.needed D.sent

六、阅读理解 阅读下面的短文,然后根据短文内容选择最佳答案。

( A )

A man made a nice talking machine .It could weigh people‟s weight .The man wanted to try the machine before he could make a lot of machines.

He put the machine into the waiting room of a station. There were always lots of people in and out.

The first one who used the machine was an Indian woman. She stood on the machine, the machine thought for a few seconds to decide

5. Last Sunday, Mum made me ________ (do) some washing at home.

6. It‟s 4:30 p.m. The students of Class 4 _______(play) football on the playground.

7. Mother asks him ______ (not forget) to lock the door when he leaves home.

8. There‟re so many wonderful clothes in the shop, so she can‟t decide which one ____________(choose).

9. They wanted to run away but we _______(catch) them in the end.

10. Linda‟s mother will come back if she ________ (finish) her work this week.

四、完成句子 按所给的汉语,用英语完成下列句子,1-4题每个空格只能填一个单词。

1. 我们应该鼓励孩子们保护动物。

We should__________ children __________ __________ animals.

2. 我愿意和我的朋友分享快乐。

I‟m ______ _______ ________my joy ________ my friend.

3. 我决定呆在家里上网,不出去购物了。

I _______to stay at home and search the Internet _______ _____ _______ shopping.

4. 在下了车之后,他们不再感到恶心了。

After they _______ ________ the coach, they didn‟t feel ______ _______ ________.

5. 西蒙的衬衫和迪克的衬衫颜色一样。

Simon‟s shirt ________________________________________.

6. 你到那儿后,请尽快给我打电话。

After you arrive there, please call me____________________________.

五、完形填空

A teacher was asking a student a lot of questions,.The teacher then decided to ask him some .

“ she asked.

The student for some time and then answered, “An important Person.”

“No.it is a battle(战役).”the teacher said.,it.Then she asked,“Who was the first president of the United States?” The student thought and thought ,but didn‟t.The teacher got very angry and shouted,“George Washington!”

“Come back!” the teacher said.“didn‟t tell you to go.” “Oh,I‟m sorry,” the student said , “I thought you the next student.”

( )1. A.wouldn‟t B.couldn‟t C.shouldn‟t D.mustn‟t

( )2. A.1ittle B.few C.a little D.a few

( )3. A.What B.Which C.Who D.Where

( )4. A.thought B.spent C.waited D.stopped

( )5. A.angry B.worried C.sad D.happy

( )6. A.find B.1ose C.do D.show

( )7. A.speak B.tell C.say D.talk

( )8. A.friend B.seat C.address D.side

( )9. A.He B.She C.I D.We

( )10. A.called B.taught C.needed D.sent

六、阅读理解 阅读下面的短文,然后根据短文内容选择最佳答案。

( A )

A man made a nice talking machine .It could weigh people‟s weight .The man wanted to try the machine before he could make a lot of machines.

He put the machine into the waiting room of a station. There were always lots of people in and out.

The first one who used the machine was an Indian woman. She stood on the machine, the machine thought for a few seconds to decide

which language to speak.

“Good morning, Madam.” It said in Indian. “Your weight is 72 kilograms. That‟s three kilograms more .If you eat more fruit and vegetables, you will be soon all right. Please have a nice day.”

The second one to use the machine was a Chinese girl .She stood on the machine and waited to hear her weight.

“Good morning, Miss.” The machine said in Chinese. “Your weight is 45 kilograms. It‟s all right for your age. Keep eating what you eat every day. Please have a nice day.”

The third one to use the machine was a very fat American woman .She thought for a long time to stand on the machine .The machine spoke quickly in English, “Good morning. Will one of you get off?”

1. The machine in this passage could .

A. speak all kinds of languages

C. weigh and talk to people

A. in the station B. tell people what they should eat D. make people laugh . B. in a train

D. in the women‟s room

B. she was a little heavier

D. to eat what she wanted to eat

B. should eat less

D. kept herself healthy

B. She liked thinking.

D. The machine didn‟t like her.

( B )

There are many kinds of animals that live in the forest, such as mice, rabbits, squirrels, bears .

Animals in the forest depend on(依靠)one another for food. Many small forest animals, such as mice, eat green plants for food, and then larger animals eat them. This is called food chain. A food chain begins with the sun. The green plants in the forest use the sun‟s energy to make food for themselves. It‟s called photosynthesis(光合作用). Mice eat flowers and seeds that forest plants produce(生产).The mice get their energy from the plants, and the mice become food for a larger animal, such as the hunting hawk. So the energy from the sun passes from the plants to the mice to the hawk.. A break(中断)in any part of this food chain means some animals may not have enough food to eat.

( )6. Photosynthesis takes place when _____________.

A. larger animals eat small animals.

B. green plants use the sun‟s energy to make food.

C. Forest animals eat the food made by green plants.

D. The food chain is broken.

( )7. What‟s the first link(环) in a food chain?

A. The sun B. The earth C. Green plants D. Forests animals

( )8. Animals sometimes cannot find enough food because _____________.

A. it‟s cloudy.

B. there is very little food in a forest.

C. Photosynthesis stops.

D. There is a break in the food chain.

( )9. What does the food chain show about living things?

A. They depend on each other.

B. Large animals eat small animals.

C. Plants with flowers are the most important.

D. Plants are more than animals.

( )10. What may be the best title of the passage? C. in the doctor‟s waiting room A. she was a little lighter C. it couldn‟t speak Indian A. should eat more 3. The machine told the Indian woman 4. The machine said to a Chinese girl that she C. had to eat more fruit A. She was healthy. C. She was too heavy. 5. How was the American woman?

A. Animals in the world

B. Food chain

C. Plants and man.

D. Animals are in danger.

七、缺词填空 :先通读下面的短文,掌握其大意,然后根据短文内容和所给首字母,在空格内填入一个适当的词,使短文意思完整。

in Russia, he ran short of (缺少)money. So he w $500. “Send the money by telegram(电报)to the bank here. ”After a week Ed began visiting thhis passport (护照)to the bank clerk(职员). “Here is n for you, ”He told him. This went o . He then phoned his brother, asking w the money was. His brother said he had sent it three weeks before. That evening Jackson was arrested (被捕)for failing to pay his bill. He t station for fifteen days.

1. t_______ 2. w_______ 3. f________ 4. s__________ 5.n________

6.o______ 7. w________ 8. w________

八、书面表达

假如Mary 是上学期来到你们班学习的一个美国女孩,请根据以下内容,向你的朋友介绍她的一些学习情况,要求条理清楚、语句通顺,字数70左右。

1、她以前在美国的一所男女混合学校读书。

2、现在对汉语有浓厚的兴趣,但学习不够用功。

3、擅长数学,每天花大量时间做练习。

4、家政是她最爱的学科,可以学习如何自己做事情。

5、她是我们班学习最好的学生之一。当我们英语学习有困难时,我们会向她求助。

6、她说在这里学习很愉快。

_______________________________________________________________________________

_______________________________________________________________________________

_______________________________________________________________________________

_______________________________________________________________________________

______________________________________________________________________________________________________________________________________________________________

_______________________________________________________________________________

_______________________________________________________________________________

_______________________________________________________________________________

_______________________________________________________________________________

初二英语寒假作业(三)

一、选择填空:

( ) 1. I can‟t speak English _______ you do.

A as well as B as good as C as better as D so well as

( ) 2. Did he and you enjoy _______ at park?

A yourselves B themselves

C yourself and himself D himself and yourself

( ) 3. The third cake is ________ of all.

A delicious B more delicious C most delicious D the most delicious

( ) 4. Nancy‟s classroom and John‟s classroom are _______.

A like B same C alike D the same as

( ) 5. Simon scores _______ points.

A fewest B least C the fewest D the least

( ) 6. We ______ friends for almost 10 years.

A have made B have been C have become D have turned

( ) 7. We should share things with others and help people ______.

A in hospital B in need C in the hospital D in bed

( ) 8. I want to tell you about my ______ friend Betty.

A true B real C really D truly

( ) 9. She is kind and never says a bad word about _______.

A anyone B everyone C someone D nobody

( ) 10. He looked so tired _________ too much hard work.

A because B because it‟s C because there are D because of

( ) 11. You can take a taxi _______ the center old Beijing to learn more about old Beijing.

A in B along C circle D around

( ) 12. I am going to see the monument _______ the People‟s Heroes at Tian‟an men Square

A of B about C with D to

( ) 13. Do you know how many _______ a cow have?

A stomach A stomaches C stomachs D stomachies

( ) 14. I wonder where ________.

A is she hiding B does she hide C she does hide D she is hiding

( ) 15. Please don‟t tell him about it. OK, I_______.

A don‟t B won‟t C am not D didn‟t

( ) 16. She is very clever______ she is good at math.

A and B but C or D because

( ) 17. My sister loves playing softball, she ______ two hours _______ every day.

A spends, to practice playing B spends, practicing playing

C spends, practicing to play D takes, to practice to play

( ) 18. _________? He is friendly and polite.

A What does he look like B What does he like

C What is he like D What is he alike

( ) 19. A horse is _______ animal. It helps people a lot.

A a useful B an useful C a useless D an useless

( ) 20. People in Britain say “film” while people in American say “_______”.

A fall B hall C elevator D movie

( ) 21. —Is ______ here? —No, Li Lei and Wang Ping have asked for leave.

A somebody B anybody C nobody D everybody

( ) 22. He spent lots of money ______ clothes.

A buying B on C to buy D A and B

( ) 23. Where are you going ______ your winter holidays?

A in B at C for D of this afternoon.

( ) 24. We went to travel with some friends of _______.

A us B ours C our D ourselves

( ) 25. My uncle arrived ______ Shanghai ______ the next afternoon.

A at… in B in… on C in… on D in…/

( ) 26. Some boys think maths is ______ English. I think so.

A much difficult than B more difficult as

C less difficult than D so difficult as

( ) 27. On the long table ________ a glass of water and some nice food.

A is B are C have D has

( ) 28. What do you often do ______ the end of the week?

A on B by C at D in

( ) 29. China is a country _________ a long history.

A have B has C with D without

( ) 30.Yesterday she ______ a red coat and looked beautiful.

A wore B put on C put up D dressed

二、 用所给词的适当形式填空:

1. Giant Panda is a _______ (danger) animal because there are only about 1,000 in the world.

2. It is important for us ______ (know) more about nature.

3. Mr. Green advised them _______ (read) more during summer holidays.

4. Some _______ (hunt) are good at hunting animals.

5. I‟m sure everyone likes _______ (peace) life.

6. The boy ran to his mother ______ (wild).

7. The story happened on a ______(fog) morning.

8. If no one ______ (buy) animal fur, people ______ (not kill) wild animals.

9. How many ______(wolf) are there in the picture?

10. Who has the ______ (key) to the doors of the classroom?

三. 完型填空:

Wang Fang is 16 years old. She helped ____1____ neighbor out ___2__ a fire. ___3__ 10th May, Wang Fang was at home alone. Suddenly, she heard someone __4___ “Fire! Fire!” She ran outside. She saw a lot of smoke from next door. She went __5___ and saw the ___6___ Mr. Sun, in the kitchen. He could not get out because he hurt his leg.

Wang Fang quickly ran back to her flat and __7___ water over her jacket. __8___ she rushed into Mr. Sun‟s kitchen ___9__ him. At last she put out the fire ___10___ a blanket and helped Mr. Sun out.

( ) 1. A her B she‟s C herself D herself‟s

( ) 2. A from B form C of D on

( ) 3. A In B At C On D From

( ) 4. A shout B shouting C to shout D shouted

( ) 5. A in B into C out D outside

( ) 6. A 79 years old B 79-years-old C 79-year old D 79-year- old

( ) 7. A pulled B poured C pushed D threw

( ) 8. A First B Next C After D Then

( ) 9. A help B to help C to save D and save

( ) 10. A use B using C used D with

四. 阅读理解:

A

The children in Miss Ling‟s class are not like most children. They don‟t watch TV. Miss Ling asked her class not to watch TV for one week. She told them that they could have more fun doing other things. The children said, “Let‟s find out if Miss Ling is right.” Instead of watching TV, they read, made things and played games with families.

The children had so much fun that they are not going to watch TV for a year. Would you like to stop watching TV?

( ) 1. The best title is ______.

A A Good TV Show B Stop To Watch TV

C Children Have Fun Without TV D Miss Ling Doesn‟t Buy Any TVs

( ) 2. Miss Ling asked her class not to ______.

A sleep B eat C read books D watch TV

( ) 3. The story says that the children _________.

A played games B went swimming C grew plants D watched TV

( ) 4. Miss Ling is a _________.

A doctor B teacher C farmer D child

( ) 5. How long are the children not going to watch TV?

A For twelve months B For 365 days

C For half a year D For three years

B

Bats are the only flying mammals(哺乳动物) in the world. They cannot see very well, so people believe they have no trouble flying no darkest nights and finding their way around very well. How can bats fly and see at night? They fly by radar(雷达).

The bat‟s radar system(系统) works the same as ships and planes do. As a bat flies through the air, he makes a sound. The sound is high enough for his message. In this way it tells the bat where the thing is.

Bats go out to look for food only at night. In the daytime they rest in some dark places.

( ) 1. The bat is ______ in Chinese.

A 蝙蝠 B 老鼠 C 蚊子 D 蜻蜓

( ) 2. The bat has no trouble flying on the darkest nights because ______.

A its eyes are good B it can‟t see well

C it can make a sound D it has a kind of radar system to help it

( ) 3. When does the bat begin to go out to look for food?

A In the daytime. B In the early morning.

C When night comes. D In the late afternoon.

( ) 4. The bat “sees” things with its _________.

A eyes B nose C mouse D ears

( ) 5. Which of the following is right?

A The bat is the only flying animal in the world.

B The bat is blind, but it can “see” its food.

C Not all bats are afraid of flying in the sun.

D The bat often comes out during daytime, too.

五. 单句改错:

1. Don‟t worry. There is little time left.

2. Because I can‟t see anything, so I never go out at night.

3. There is going to have a concert next Sunday.

4. What a beautiful weather it is today!

5. Tom and Jack are all students.

6. Which is much interesting, Shanghai or Beijing?

7. How long do you go home? Once a week.

8. Your mother must keep health.

9. What is the weather like yesterday?

10. The floor is all wet. Please mop up it.

11. They have more rains this year than last year.

12. She crashed her car on a wall.

13. He was died in a car accident.

14. A boy fell down a tree and broke his left leg.

15. Look! His clothes are on a fire.

六. 完成句子;

1. 吴先生邀请我参加他们学校去世界公园的旅游。

Mr. Wu invited me _____ _____ _____ their school trip to the World Park.

2. 利奥的铅笔盒和克丽斯廷的铅笔盒尺码一样大小。

Leo‟s pencil box is ______ _____ _____ as Christine‟s pencil box.

3. 朱莉同意和他们去。

Julie agrees _____ _____ ____ them.

4. 长大后,贝蒂想当歌手。

Betty wants to be a singer when ______ ______ ______.

5. 她明亮微笑的双眼令她看起来真的很漂亮。

Her bright, smiling eyes _______ her ______ _______ pretty.

____________ ____________ ____________ ____________ ____________ ____________ ____________ ____________ ____________ ____________ ____________ ____________ ____________ ____________ ____________

初二英语寒假作业(四)

一、单项选择

( )1. It‟s necessary ______ us ______ read English every day.

A. of, to B. for, to C. for, for D. to, to

( )2. She likes _____ her joy ______ her friends.

A. share, with B. sharing, with C. share, for D. sharing, for

( )3. Do you think British students spend___ time ___ their homework than Chinese students?

A. less, doing B. less, to do C. fewer, doing D. fewer, to do

( )4. It was getting dark. There were ______ people in the street.

A. less and less B. fewer and fewer C. less and fewer D. fewer and less

( )5. He finds _______ to make a paper ship.

A. it easy B. that easy C. it easily D. that easily

( )6. We have got everything ready. So we need _______.

A. something else B. else something C.nothing else D. else nothing

( )7. The teacher told us to leave school early ______ the snow got worse.

A. but B. so C. because D. and

( )8. If Jim ______ free tomorrow, I‟ll ask Tom instead.

A. isn‟t B. won‟t be C. won‟t D. doesn‟t

( )9. ----Did you buy the camera yesterday?

----No. I didn‟t buy it ____ my aunt would give me one.

A. until B. because C. if D. before

( )10. The teacher is coming. Stop ______ and keep ______.

A. to talk, quiet B. talking, quiet C. to talk, quietly D. talking, quietly

( )11. Our government provided food and money____ the people in the disaster areas.

A. for B. with C. to D. on

( )12. There will be a strong ______ tomorrow and it will be quite ______.

A. wind, wind B wind, windy C. windy, wind D. windy, windy

( )13. The news report is about ______ a month ago.

A. what he happened B. what he happened to

C. what happened him D. what happened to him

( )14. The capital of the USA is_______.

A. London B. New York C. Bangkok D. Washington DC

( )15. --- My brother fell off his bike and hurt his legs. --- ______.

A. That‟s too bad B. He is too careless

C. He should be careful D. I am sorry to hear that

( )16. The land is _______ after a heavy rain.

A. much weter B. much wetter C. more weter D. more wetter

( )17. The sunlight came in ______ the window.

A. through B. across C. in D. with

( )18. What did you do in the garden? I watched John ______ his bike.

A. to mend B. mended C. mend D. mends

( )19. The big box is ______ heavy and I can‟t carry it.

A. much too B. too much C. so much D. very too

( )20. Daniel thinks climbing is ______ diving.

A. so excited as B. as excited as C. so exciting as D. as exciting as

二、完型填空

It was December 25, 1972. Marie, a 13-year-old Australian girl, was very happy. It was and Marie‟s mother made an unusual cake. She put four small coins(硬币) . mother didn‟t notice that.

. She coughed and couldn‟t speak. Six weeks later she felt went to the hospital.

(X光). Marie‟s mother asked the doctors, “Why can‟t Mariedoctor said, “We don‟t know. Maybe she will speak again. Maybe she won‟t. We‟re, but we can‟t help her.”

For twelve years Marie didn‟t speak. SheOne day when Marie was 25 years old, she got ill. She began to cough. She coughed up a lot of blood(血) small and black from her throat(喉咙). What was it? Marie didn‟t know. She took it to the hospital. A doctor said, “This is a coin.”

The doctor told Marie, “I

( )1. A. party B. Christmas C. holiday D. winter

( )2. A. the cake B. her pocket C. Marie‟s bag D. the box

( )3. A. friends B. health C. luck D. life

( )4. A. teacher B. brother C. daughter D. family

( )5. A. began B. finished C. did D. made

( )6. A. paid B. found C. missing D. put away

( )7. A. tired B. home C. back D. ill

( )8. A. worse B. terrible C. better D. lonely

( )9. A. still B. also C. yet D. often

( )10. A. looked for B. looked over C. looked after D. looked up

( )11. A. speak B. listen C. tell D. hear

( )12. A. sure B. busy C. worried D. sorry

( )13. A. put up B. sent up C. grew up D. got up

( )14. A. something B. nothing C. anything D. everything

( )15. A. sure B. tell C. ask D. think

三、 阅读理解

(A)

Everyone needs friends. There is an old saying: Friends are God‟s way of taking care of us. But how do you find real friendship and keep it?

“The Care and Keeping of Friends” by American author Sally Seamans tells young students some smart ways to find friends. Sally says finding friendship is just like planting a tree. You plant the seed and take care of it to make it grow.

First, you should choose a friend. What makes a good friends? It is not because a person has money or good looks. A good friend should be kind and patient. For example, if you have a bad day, a good friend should listen to your complaints(抱怨) and do their best to help. To make a friend, you cannot be too shy. You should make each other happy and share your lives.

But things cannot always be happy. Even the best friends have fights. What should you do when you have a fight with your friend? You have to talk to them. When there is no one around, have an honest talk. If he or she doesn‟t want to talk, you could write a letter.

Sally says there are three steps to being friends again: Tell him or her how you are feeling; say what your friend has done wrong, and explain why you did this or that. The book also has advice on some small but important things like celebrating your friends‟ success. Even if you haven‟t had a real friend before, you will start to think of having one if you read this book. Because the book tells that friendship is the most important thing in your life.

( )1. The writer of the text really wants to tell us________.

A. life is great if we have friends B. an old saying

C. Sally Seamans is an American writer D. God‟s way of taking care of us

( )2. “The Care and Keeping of Friends” is_______.

A. the word of the God B. a way of making good friends

C. the name of a book by Sally D. a story from a newspaper

( )3. A good friend should_______.

A. have a lot of money B. have good look

C. be kind and patient D. listen to you

( )4. Good friends will_______.

A. always be happy B. do their best to help you

C. never do wrong D. not have a fight

( )5. What can‟t you do if your good friend has done wrong?

A. Write a letter to him. B. Have an honest talk with him.

C. Tell him or her your feeling. D. Have a fight with him.

(B)

Can you make animals work for us? Some scientists think that one day we can teach animals to do a lot of things for people. In a film shown on TV, you may see elephants, monkeys, tigers or some other animals are always given a little food to eat after they have done something. Scientists say that people can teach many different animals to do some of the easy work if they know they will get something to eat. We all know elephants can carry large logs(圆木),and dogs can look after houses.

And we even teach animals to work in factories. In America, for example, people have used apes to help make cars and scientists think that those large monkeys may drive trains one day. Now people are trying to get them to do the same thing that man does.

( )6.Scientists think animals can help people to do something .

A. if they know they will get food after doing something

B. because animals are so clever.

C. if they know the work is easy enough

D. because most animals are friendly to man

( can do some heavy work because they are so strong.

A. Tigers B. Elephants C. Monkeys D. Dogs

( )8.As we know, dogs can our houses.

A. build B. clean C. look after D. look at

( )9.Scientists are planning to make big monkeys to .

A. make trains B. carry logs C. drive trains D. work at school

( )10. If we teach animals carefully, .

A. some animals can do the same things as a man does

B. all the animals can do some work for us

C. many animals can do all the work as we do

D. we may get many animals work without giving them food

四、词形变换

1. Don‟t forget _______ (lock) the door when you leave home.

2. Do you know what________(do)next?

3. The new tie makes him _______ (look) smart.

4. I will give it to her as soon as she ____(come) back next week..

5. Our teacher told us _______ (not get) to school late next time.

6. Do you mind ______(close) the window?

7. If you want to learn English better, you should practise _______ (speak) English often.

8. Both the twins enjoyed ______(they) at the party.

9. My pet lives_______(comfortable)in his cage.

10. Tim was ________ (trap) in a dark place when the earthquake stopped.

五、完成句子

1.你要保守秘密,否则这会使他不高兴。

You should _______ .

2.今天完成这项工作是不可能的。

It is __________ _________ __________ the work today.

3.我们应该鼓励孩子们保护动物。

We should __________ children __________ animals.

4. 他没有你强壮。

He isn‟t_____ ______ _____ you.

5.我们需要更多人来认识到湿地的重要性,并且采取措施来保护野生生物。

We need more people to know the _____of the wetlands and___ ___to protect wild animals.

六 、书面表达

请根据下列内容,写一篇短文,要求要点全部写出。

昨夜,一场可怕的洪水袭击了我村,冲走了许多房屋。洪水到来时我和我父母仍在睡觉。我们急忙到屋外,看到四周都是水。我们将餐桌从屋内移到了屋外,但它的大小只够两个人坐在上面。屋前有颗大树,于是我快速爬上了那颗树并在上面待了好几个小时。

___________________________________________________________________________________________________________________________________________________________________________________________________________________________________________________________________________________

初二英语寒假作业(五)

一、单项选择

( )1. He never tells lies, so he is _____ honest boy. It‟s not good to be _____ dishonest one.

A. a , an B. an , a C. /, a D. an ,the

( )2. When his father London, he went to see the Big Ben.

A. arrived B. reached C. got D. arrived to

( )3. Let‟s enjoy the song “Yesterday once more”. It sounds

A. bad B. sadly C. well D. wonderful

( )4. __________ nature reserves _________ us learn more about protecting animals.

A. Studying , helps B Studies , to help C. Study , helping D. Studying, help

( )5. Red-crowned crane is a kind of bird.

A. endanger B. danger C. endangered D. dangerous

( )6. I am in places of in Nanjing.

A. interesting, interest B. interested, interest

C. interested, interesting D. interesting, interested

( )7. “X plus Y is more than Fifteen percent.” This sentence can also write as “_________”

A. X+ Y>15% B. X+Y≥15% C. X+Y=15% D. X+Y<15%

( )8. The floor is quite wet, my teacher asked us _________ quickly.

A. to mop up it B. mopped it up C. to mop it up D. mop it up

( )9. If we keep the land, giant pandas will have to live.

A. to take, anywhere B. taking, anywhere C. to take, nowhere D. taking, nowhere

( )10. If someone shows good manners to others, he is a ________ person.

A. impolite B. polite C. rude D. dispolite

( )11. He wears small, round glasses and they make him _________ smart.

A. to look B. to be C. looks D. look

( )12. He was all wet when he got home, because he his umbrella in the underground.

A. took B. forgot C. left D. brought

( )13. The number of cranes in the world is getting these years.

A. less B. much C. little D. smaller

( )14. The Taiwan Earthquake killed people.

A. two thousands B. two thousands of

C. thousand of D. thousands of

( )15. Which of the following is not a natural disaster?

A. A car crashed into the tree B. A big flood

C. A storm D. The Taiwan earthquake

二、完形填空 lovely birds. will soon catch it.

Seagulls swim well, but they don‟t often dive for fish. They are also good at flying _____said she didn‟t like the noise they made.

( ) 1. A. watch B. to watch C. watching D. watched

( ) 2. A. understood B. learned C. found D. thought

( ) 3. A. throw B. take C. send D. pull

( ) 4. A. with B. to C. for D. of

( ) 5. A. below B. above C. on D. in

( ) 6. A. to come B. came C. coming D. comes

( ) 7. A. help B. beat C. put D. kept

( ) 8. A. hit B. arrived C. got D. touched

( ) 9. A. water B. land C. sky D. ground

( ) 10. A. eyes B. feet C. legs D. wings

三、阅读理解

(A)

Many animals do strange things before an earthquake. This news may be important. Earthquakes can kill people and knock down homes. The animals may help to save lives.

Some animals make a lot of noise before an earthquake. Farmers have told about this. Dogs that are usually quiet have started to bark. Horses on farms have run around in circles. Mice have left their holes and run away. Cows have given less milk.

In a town in Italy, cats raced down the street in a group. That happened only a few hours before an earthquake. In San Francisco, a man kept tiny pet frogs. One Sunday, the frogs jumped around more than ever. They make loud noises, like bigger frogs. That night, an earthquake struck the city.

People want to know when an earthquake is coming. Then they could get away safely. Right now, there is no sure way to know ahead

of time. Maybe the best idea is watching the animals.

( )1. This passage is mostly about .

A. how animals act before an earthquake B. how an earthquake starts.

C. how mice leave their homes. D. how animals live every day

( )2. Before an earthquake, quiet dogs A. ran away B. started to bark

C. climbed trees D. jump into the water

( )3. Before an earthquake, the frogs .

A. sang B. jumped into the water

C. left their homes D. jumped around a lot

( )4. There have been earthquakes in A. most countries B. Taiwan, China

C. Italy and San Francisco D. Chicago and Spain

( )5. People want to A. be in earthquake B. find out early about an earthquake

C. run around in circles D. follow the animals to run quickly

(B)

Young people can have problems with their minds. Some students become worried because they have to study very hard. Others have trouble getting on well with people like their parents and classmates.

Lu Jian, a student from Henan, could not understand his teacher and was doing badly in his lessons. He became so worried about so that he started to cut his finger with a knife.

Another student, 16—year—old Liu Fang from Guang dong, was afraid of exams. She got very worried in one, and when she looked at the exam paper. She couldn‟t think of anything to write.

A recent report from China Daily says about 14% of Beijing teenagers have mental problems. Their troubles include worried and very unhappy. And having problems in learning and getting on with people. Many students who have problems won‟t go for advice or help. Some think they will look stupid if they go to see a doctor. Others don‟t want to talk about their secret.

Gao Xiao, an expert(专家) on teenagers from Beijing Anzhen Hospital has the following advice for teenagers:

*Talk to your parents or teachers often.

*Take part in group activities and play sports.

*Go to see a doctor if you feel unhappy or unwell.

( ) 6. The students who often become worried or have trouble getting on with others may have _________________.

A. mental problems B. a headache C. knives with them D. no parents

( ) 7. Lu Jian cut his finger with a knife because______________.

A. he was afraid of his teacher. B. he wanted to frighten his parents

C. he was so worried about his studies D. his finger was badly hurt

( ) 8. Liu Fang‟s problems happened when __________________.

A. she studied very hard B. she had exams

C. she talked with her parents D. she thought of something

( ) 9. Students who have problems won‟t ask others for help because _______________.

A. they won‟t let others think they are stupid

B. they don‟t think doctors can help them.

C. they don‟t want to tell their secret to others

D. both A and C.

( ) 10. Gao Xiao‟s advice tell us that ___________________.

A. it‟s better for the students who have mental problems to join others

B. it‟s unnecessary for them to be with others.

C. only group activities and sports can help them.

D. teachers and parents can‟t do anything about mental problems

四、填空与改错

A)根据括号里所给的汉语、英文解释、首字母及句意填入合适的单词

1. Let‟s go bird watching in the (森林).

2. The girl __________ (尖叫) when she saw the snake.

3. Is it

5. Do you know the 原因) why he didn‟t come to your party?

B)根据句子意思,用括号所给单词的适当形式填空。

1. It is _________ (correct) to hunt red-crowned cranes.

2. If you still do your homework _________(care), you will lose the exam again.

3. The members of our club encourage hunters_________(not kill) rare animals.

4. If it 5. Look! The bus __________ (come).

C)下列各句的A、B、C、D中各有一处错误,在不改变句子意思的前提下,将正确答案写在题后的横线上。

(A) (B) (C) (D) (A) (B) (C) (D) tomorrow.”

(A) (B) (C) (D)

4. Don‟t make so (A) (B) (C) (D) (A) (B) (C) (D) D

2. you have enough food when you go camping.

3. Last time, after the big earthquake, Sandy ______________.

4. Is your pet dog good at 5. If you play football in the busy street, you _________________.

五、句型转换

1. People were frightened to look at each other. (同义句)

People looked at each other _____________.

2. It‟s important for us to read English every day. (同义句)

We must know English every day.

3. Mr. Wu warned him: “Don‟t run in the street.”(同义句)

Mr. Wu warned him .

4. It‟s 划线提问) the weather today?

划线提问)

六、书面表达

A)翻译

1. 狐狸有好的视觉,听觉和嗅觉。

Foxes have 2. 人们改变湿地来为农场腾出更多空间。

People changed the wetlands farms.

3. 如果你每天练习,你就将很好听地唱歌。

If you practice every day, you _____________ songs _______.

4. 人们逃向四面八方。 People run .

5. 当我听见一个巨大的声响,一阵恐惧涌上心头。

B)周末你去北京野生动物园参观。请根据你所观所感,写一篇短文,要点如下:

1. 有很多游客,他们给猴子喂食,和老虎玩耍。

2. 动物和人很友好,我们应该善待动物。

3. 保护环境非常重要,动物园内不要随地扔垃圾。

4. 知道了野生动物保护的重要性。因为保护动物就是保护我们自己。

____________________________________________________________________________________________________________________________________________________________________________________________________________________________

初二英语寒假作业(六)

一、选择填空

( )1. Which is the cheapest?

( )2. My father was born_______1964. He is forty years old.

A. at B. on C. in D. to

( )3. Animal World is the most interesting TV________.

A. play B. sport C. team D. program

( )4. Please give________ a slice of bread, Mom. I‟m very hungry.

A. I B. my C. me D. myself

( )5. Paul likes to help others. He is very________.

A. happy B. kind C. quiet D. funny

( )6. — ________ water do we need? — A bottle.

A. How many B. How long C. How often D. How much

( )7. My mother _________lots of things and took the subway back home yesterday.

A. buys B. bought C. is buying D. is going to buy

( )8. — Could you please go sightseeing with us this Sunday?

— _________. I have to finish my homework.

A. Yes, sure B. I‟m sorry C. Excuse me D. Good idea

( )9. I don‟t like the weather in Xi‟an. It‟s dry in summer. It______ rains.

A. seldom B. usually C. always D. often

( )10. We‟re going to the Circle Theater in John‟s car. You can come with us _______ you can meet us there later.

A. but B. and C. or D. then

二、完形填空。

( )11. game B. video C. day D. sale

( )12. put B. entered C. jumped D. stayed

( )13. played B. did C. bought D. took

( )14. A. in the room B. in the yard C. at school D. at the cinema

( )15. A. exciting B. interesting C. relaxing D. boring

(B)

What is a museum? A museum is a good place to keep and beautiful things. A museum may be a place to learn about science. A museum can be a place to the history about art or Indians or animals.

What is inside a museum? Some museums have old cars and airplanes. Many museums have pictures and statues. and old bones. One museum even has a coal mine inside!

Many cities have museums. Some very small towns have museums, too. Indianapolis has a free museum for children. Children do not have to to get in. They like to look at the dinosaur bones(恐龙化石). They see a white bear ten feet tall there.

On Saturdays, Does your town or city have a museum?

( )16. A. new B. old C. bad D. fun

( )17. help out B. hang out C. try out D. find out

( )18. Others B. The others C. The other D. Another

( )19. go B. like C. pay D. learn

( )20. boys B. girls C. children D. parents

三、阅读理解

(A)

Joe 正在介绍自己的像册。请你为每一幅照片划到相应的描述,并将字母填入相应的( )内。

Hi! My name is Joe Williams. I‟

m 15 and I come from Atlanta. This is my photo album.

A. These are my grandparents, Timothy and Mary. They come over to our house almost every Sunday for dinner, and they always help with the cooking. My grandpa was a teacher, but now he‟s retired. He helps me study when I have exams. My grandma loves playing with my little sister.

B. Here I‟m with my friends. We‟re in the came class and we love basketball. I‟m tall, so I play on the school team. My favorite team is the Chicago Bulls.

C. After school I usually have lots of homework and very little free time. My favorite hobby is playing the guitar. And I also play in the school band. We aren‟t very good, but we keep practicing.

D. These are my parents, Alex and Beth, and my little sister Jenny. She‟s only five. She loves picnics in the park because she can run around and she often feeds animals. She loves animals very much. She says she wants to be a doctor for animals when she grows up.

(B)

Jenny mixed up the letters with the replies. Read the three letters and the replies A~C, match each letter with the correct reply. 你能将Jenny的回信交给不同的来信者吗?请将字母填入相应的( )内。

( )28. What is Mary‟s problem?

A. She did badly in some important exams.

B. She worries too much about her exams.

C. She can‟t do her homework very well.

( )29. What shouldn‟t Mary do?

A. Find more time to relax than before.

B. Do a lot of exercises and think positive.

C. Drink coffee or tea before going to bed.

( )30. Why don‟t Nina‟s parents want to buy her a mobile phone?

A. It is too expensive and they can‟t pay for it.

B. It isn‟t very good for Nina‟s health.

C. They‟re afraid Nina will use it a lot.

( )31. If Nina gets a mobile phone, she should __________.

A. pay the bills all by herself

B. ask her parents to pay the bills

C. ask her parents for more pocket money

( )32.Why doesn‟t Jim spend much time with his father?

A. Because Jim has a lot of homework to do.

B. Because Jim‟s father works a lot every day.

C. Because Jim‟s father got a new job in another city.

(C )

阅读短文后选择正确答案.

As teenagers, you have many dreams .These dreams can be very big, such as winning the Nobel Prize, or they can be small. You may just want to become one of the ten best students in your class.

Once you find a dream, what do you do with

it? Do you ever try to make your dream real?

“Follow Your Heart”by Australian writer

Andrew Matthews tells us that making our dreams

real is life‟ You may think you‟re not very good at some

school subjects ,or that it is impossible for you to

become a writer. These kinds of thoughts stop you

from getting your dream, the book says.

In fact, everyone can make his dream come true.

The first thing you must do is to remember what your

dream is.

Don‟t let it leave your heart. Keep telling

yourself what you want. Do this step by step and

your dream will come true faster because a big dream is,

in fact, many small dreams.

You must also never give up your dream.

There will be difficulties on the road to your

dreams. But the biggest difficulty comes from yourself. You need to decide what is the most important. Studying instead of watching TV will lead to better exam results, while saving five yuan instead of buying an ice cream means you can buy a new book.

As you get closer to your dream, it may change a little. This is good as you have the chance to learn more skills and find new interests. ( )33.“Follow Your Heart”is .

A. the first thing you must do to make your dream real

B. the most important dream to have for teenagers

C. the name of a book by Andrew Matthews

D. the name of a famous Australian writer

( )34.If you want to make your dream real, you should .

A. remember what you dream is first

B. keep telling yourself what you want

C. never be afraid of the big difficulties

D. try to start with all the things above

( )35. From the text, we know the word challenge means .

A.变化 B.挑战 C. 决定 D. 态度 ( )36. What‟s the best title for the text?

A. Keep your dream in your mind. B. How to find your real dream C. Never stop dreaming of success. D. Teenagers have many dreams.

四、语言综合能力运用

一、很多同学都有自己的特长,如果有机会你将在哪里展示这些特长?你将如何展示这些特长呢? (A) 请根据提示,将所给单词、词组分类;只写字母即可。

1. Our talents are ___, ____,____,_____,_____.

2. We‟ll show our talents in/on / at the ____,_____,____,____,_____,_____. 3. We can use the ____,____,______,_____,_____ to show our talents. (B)根据自己的实际情况,选择一项内容填写。 1. If you have a talent, how do you show your talent?

I am good at , I‟ll To show my talent.

2. If you don‟t have a talent, what are you going to do?

I am not good at

二、周末就要到了,让我们为周末做一些准备吧!

(A)Nancy的父母打算周末邀请朋友共进晚餐,妈妈想让Nancy帮助做一些准备,请根据图片补全文段。 (4分)

Nancy,

I‟? ‟ Thanks Mom

(B)本周日你邀请朋友参加你的生日聚会。你想请妈妈为你做哪些准备 — 烹制美味食品? 买些饮料、小吃?为喜欢音乐的朋友准备些CD?还要买生日蜡烛(candles)吗?

你可以依照上面的文段,给母亲留言,写出你的请求。(40~50words) Dear Mom,

It will be my birthday this Sunday. I invited my friends to take part in my birthday party. .

三、寒假就要到了,你父母想带你外出旅游。请根据下表提供的信息,在三个旅游地中,选择一个你最想去的地方和合适的交通方式,写出一篇旅游计划,并简单说明理由。

初二英语寒假作业(七)

一、单项填空

1. look! The traffic light is red. You __________ stop. A. must

B. may

C. can

2. —What does the sign mean?

—It tells us ___________ on the road.

A. to ride B. not to ride C. don‟t ride 3. —The bus is too __________ to get on. Shall we wait for the next one? —All right.

A. clean B. empty C. crowded 4. —A stranger came to my room ___________ I was sleeping last night. —Really! Were you scared(恐惧的)?

A. while B. after C. before

5. —Will the girl keep the smallest box of chocolates for _____________? —I guess she will choose it, for she is very kind. A. her B. herself 6. —Did your father buy you a birthday present?

C. yourself

—Yes. He bought me a toy elephant, _____________ I don‟t really like it. A. and B. so C. but 7. —Will there be ___________ fine days in Beijing next year? —Yes. The sky will become clearer.

A. fewer B. more 8. —Wow! So many people. Let‟s go to the front. A. jump, will be 9. —Angry? What‟s up?

B. jump, are

C. less

—Oh, no. If you __________ the queue, other people ____________ unhappy.

C. will jump; will be

—Yes. Misha ____________ my mobile phone without telling me. A. took back B. took off 10. —Maybe you can take a taxi to the bank. It‟s faster. —_______________ A. Thanks. I will. B. Yes, please.

C. Good idea. Let‟s hurry.

11. _________ her mother ____________ her father lives with the little girl. They go to work in a big city. A. Both, and B. Either, or 12. —Look! Lin Tao is catching up. ________________

C. Neither, nor

C. took away

—Come on! Lin Tao. A. Is he running fast? B. How fast he is running!

C. How fast is he running?

13. —What‟s wrong, Jenny? You look unhappy.

—Betty has the same clothes ____________ I do. I want to be different. A. as B. with C. to 14. —I‟m ___________ the dog downstairs. It often makes too much noise. —Why don‟t you talk to your neighbour, the dog‟s owner? A. surprised at B. worried about 15. —What ____________ at nine o‟clock last night, sir? —Why? What happened? A. were you doing

二、完形填空

Elena was not a good student. Her head was _____16_____ most of the time. She wanted to listen in class and to study, but other things were more important: her boyfriend, her clothes, her hairstyle, and _____17_____. Every time she tried to concentrate(集中)on her lesson, her mind was gone.

One day, her English teacher, Mrs. Frederickson, told the students there was going to be an important test. She reviewed all week. Where was Elena? She _____18_____ to be there, but her mind was not.

“Did you copy the questions? Did you do the exercises? Did you write your homework? Elena, _____19_____! You‟re going to fail(not pass)the test,”warned Mrs. Frederickson.

The day of the test arrived. Elena didn‟t study very much, and she didn‟t know _____20_____ questions. She tried to guess but soon stopped.

Next to Elena sat Ivan, a very _____21_____ student. Elena didn‟t want to fail the test, so she decided to cheat. She knew it was wrong, but she had no _____22_____ of passing the test. She began to copy all of Ivan‟s answers.

Ivan noticed Elena cheating and was very _____23_____. Quickly he changed all his answers so that they were not correct. So did Elena. Then, _____24_____ his paper, he quickly changed his answers back to the way they _____25_____. Elena wasn‟t quick enough and the bell rang. Ivan turned to Elena and laughed.

“Honesty(诚实)is the best policy,”he said. “Now all your answers are wrong for sure.” 16. A. in use 18. A. seemed 20. A. some 21. A. helpful 22. A. hope 23. A. pleased 24. A. taking 25. A. did

B. in the clouds B. subjects B. tried B. wake up B. any B. key

B. polite B. angry B. passing B. might

C. in the dark C. television C. wanted C. make up C. much C. serious C. idea C. sad

C. hiding C. could

D. in the past D. school D. liked D. look up D. many D. careless D. wish D. interested D. holding D. were

17. A. exercises 19. A. come up

B. did you do

C. are you doing

C. fed up with

三. 阅读理解

A

Dan lives in Maine. He has very poor eyesight(视力)for years. He now has a guide animal to help him. The guide animal is called Cuddles. Cuddles is really a small horse. She is fully grown and just 22 inches tall. Cuddles gives Dan the help he needs and also gives him love and comfort(安慰).

Dan first heard about such horses on a television show, He found out about two horse lovers named Don and Janet Burleson. They train small horses to lead(引导)blind people.

The Burlesons got the idea for guide horses during a trip to New York City. While riding in a carriage(马车)pulled by large horses, they thought about their small pet horse Twinkie. She was friendly, clever, and kind. They wanted to know if they could train Twinkie to help a blind person. When the Burlesons returned home, they began to train Twinkie, like training dogs. Twinkie learned very well, so they decided to train other little horses.

Now the Burlesons start training these special horses when they are six months old. The training takes about a year. Each horse has a lot to learn during that time, such as to lead a blind person safely, to ride in a car and on a bus, to climb stairs and to ride in lifts. These are not places where a horse usually goes. It takes time and practice for the horse to get used to being inside buildings.

26. Dan has ____________ to help him. A. a dog C. Don

B. a horse D. Janet

B. One year.

27. How long does it take to train a guide horse? A. Six months. C. Two years.

D. Six years.

B. trained dogs

D. moved to New York City

28. Before the Burlesons started training many horses, they ___________. A. trained Twinkie C. visited Dan

29. Look at the chart below.

Which one can you put in the empty box? A. Eat grass

B. Have good eyesight C. Live 30 years D. Climb stairs

B

I met a friend yesterday but had forgotten his name. I know that he had forgotten mine, too. I felt very embarrassed(尴尬的)at that time and didn‟t know what to do. My question is: What would you do if you met this embarrassing situation(情况)?

__________________________________________________________________________

Bring in a third person. Pretend(假装)you know that person. The third person will(with luck)be too polite to say that he has never met you. You say to the third person, “I‟m sorry, but I‟ve forgotten your name.”This person will then introduce himself. You can then introduce yourself. Your friend will now introduce himself. Now you have got your friend‟s name.

Jane,

Lincoln, England

--------------------------------------------------------------------------------------------------------------------- Move to France. “Bonjour monsieur / madame”is OK if you don‟t know someone really well.

Joseph,

London, England

---------------------------------------------------------------------------------------------------------------------

The orchestra conductor(交响乐指挥家)Sir Thomas Beecham told this story:After conducting a concert, a woman started to talk with him, but he didn‟t know her. When she talked about her brother, Beecham saw his chance. He asked about her brother‟s health and asked if he was still doing the same job. “Oh, yes,”she said, “he is very well and is still king.”

Mark,

Oxford, England

---------------------------------------------------------------------------------------------------------------------

You could try the Samoan(萨摩亚人)method. There, people say hello to one another with their own names. So Dick Collins would say in Samoa, “Hi, Dick Collins!”And Jonathan Green would answer,“Hi, Jonathan Green.”

Mike,

County Cork, Ireland

30. What would Jane do if she met this situation?

A. She would pretend to know her friend and talk with him. B. She would get her friend‟s name by bringing in a third person. C. Jane thinks it‟s OK to ask someone‟s name straight. D. She would let a third person ask her friend‟s name.

31. Why did Sir Thomas Beecham ask something about the woman‟s brother?

A. Because he wanted to know how her brother was. B. Because her brother was one of his good friends. C. Because he wanted to know who she was. D. Because her brother was king.

32. Which of the following sentences is correct?

A. The report tells us about all kinds of embarrassing situations. B. The passage is most probably from a letter. C. It‟s very important to remember names in France. D. Sir Thomas Beecham spoke to the sister of a king.

C

Oseola McCarty left school in the sixth grade to go to work. For most of her 87 years, she took in wash from people of Hattiesburg, Mississippi. She never married, never had children and never learned to drive. All she ever had was work, which she saw as a blessing. Too many other black people in rural(乡下的)Mississippi didn‟t have even that.

McCarty spent almost nothing, living in her old family home, and tying her old Bible(圣经)with tape. Over tens of years, her savings-mostly dollar bills and change-grew to over $ 150,000. “More than I could ever use,”McCarty said.

So she is giving her money away to black students at the University of Southern Mississippi, where students have to pay about $ 2400 a year. “I wanted to share my savings with the children,”said McCarty, whose only regret(遗憾)is that she never went back to school. “I believed the money would do them a lot than it would me.”

The college has already awarded(奖励)$ 1000 in McCarty‟s name to Hattiesburg honors (荣誉)student Stephanie Bullock, 19. Though Bullock was in her fourth year and had high grades, her family couldn‟t afford four years of college. Then she learned that the university was giving her $ 1000 in McCarty‟s name. “It was a surprise and an honor,”Bullock said.

Bullock visited McCarty to thank her personally. Now she often visits McCarty, and drives her around. That fills a space in the woman‟s home that has been empty for years.

While McCarty does not want a building or a statue(塑像)in her honor, she would like one thing: to be at the graduation(毕业)of a student who made it through college because of her present. 33. The underlined sentence means ____________. A. she cut the toes off to fit the shoes B. she wore the shoes out to fit her feet C. she made the shoes right for her feet D. she threw away the thoes to buy right ones 34. From the passage, we know _________________. A. All the black students need money.

B. McCarty would like to be at students‟ graduation. C. Stephanie Bullock is good at her subjects. D. Bullock‟s visits disturbed McCarty‟s quiet life. 35. Why does McCarty give away her money? A. She got less education and wants to be helpful. B. She has no family and lives by herself. C. She made and saved a lot of money. D. She wants people to remember her forever.

四. 口语交际

从B栏中选出恰当的句子与A栏中的句子组成对话。

A

Patient:My head hurts badly, Doctor.

Doctor:Open your mouth and say“Ahh”. Hm„ _____36_____ Patient:Yes, but my temperature is all right. _____37_____

Doctor:You‟ve got a cold. _____38_____ Take this medicine twice a day and drink more water. Patient:____39_____

Doctor:It doesn‟t make any difference. Patient:Thank you.

Doctor:_____40_____ You‟ll be better soon if you can sleep well. Patient:Thanks a lot.

B

A. Nothing serious.

B. Shall I take it before of after meal?

C. Did you take your temperature at home?

D. What‟s wrong with me?

E. That‟s all right.

五. 语言知识运用

(一)请用所给词和词组的正确形式填空

41. —I would like to buy a pair of shoes for my son.

—What ____________ do you want?

42. —Could you help me ______________ this big wooden box? It‟s in the middle of the road.

—It‟s my pleasure.

43. —April Fool‟s Day is time to _______________________ others, isn‟t it?

—Yes. But remember not to hurt other people.

44. —What happened to Roy?

—He was in trouble but he _______________ all help yesterday.

45. After a long cold walk I was _______________ for a hot drink.

46. —Sorry, I didn‟t do well. I...

—You ________________________, and that‟s important.

47. —___________________________, Jack is the winner of the race. He looks really weak.

—You can‟t judge(判断)a book by its cover.

(二)根据中文意思完成句子。

48. 直到妇女儿童都登上救生艇,男人们才离开船。

The men ___________ the ship __________ all the women and children got into the lifeboats.

49. 他的英语说得和美国人一样好。

He _______________ English ________________ an American.

50. 当我在河边散步时,我看到有人正往河里乱丢垃圾。

When I _______________ along the river, I saw _______________ waste into it.

51. 他发现算出这道物理题不容易。

He __________________ it not easy ______________ this physics problem.

52. 我们一定要制止那些孩子在墙上乱涂乱画。

We must _______________________________________ on the wall.

六. 阅读与表达

When you go shopping, you get at least one plastic bag to carry what you bought. They are so common(普遍)that we don‟t often give them a second thought. Coles Bay, a small town in Tasmania, Australia, is trying to be the country‟s first plastic bag-free town. Since April 28, the small town‟s population(人口) Ben Kearney, a local businessman who supports(支持)the bag ban(禁令), said it would help people cut down on the use of plastic bags. “Most of the business here comes from the tourism and the good environment(环境)is important and necessary, so people are pretty supportive,”he said.

Every year Australians use more than 6 billion plastic bags. It takes years for the bags to biodegrade(生物降解). They kill about 100,000 seabirds and animals, which mistake the bags for food, every year.

Some Australian supermarkets have begun using fewer and fewer plastic bags in the past few months. Environmental groups are pushing for a plastic bag fee(收费)like that in Ireland. There, since plastic bags cost 10 pence(about 1 yuan)each, their use has been cut by 90 per cent.

阅读短文,回答下面的问题。

53. What kind of town is Coles Bay going to become?

______________________________________________________________________

54. How many people are living in the town?

______________________________________________________________________

55. Does the word“reusable”mean“that can be used again and again”?

______________________________________________________________________

56. Why does Ben Kearney support the bag ban?

______________________________________________________________________

57. After reading the passage, try to find another way to cut down on the use of plastic bags.

______________________________________________________________________

七. 书面表达

“五一”假期中,你作为一名志愿者(volunteer)来到公园里做好事,请记述你所做的事情。

根据题意,写出意思连贯,符合逻辑的短文。字数不少于50字。

所给英文提示词语供选用。

take pictures,

stop,

step on the grass, old people, pick flowers, handicapped(有障碍的,残疾的)

初二英语寒假作业(八)

一、单项选择

1. Mary is Australian, ____________ she lives in Germany.

A. but B. or C. and

2. —____________ do you write an English composition?

—Sometimes.

A. How long B. How often C. How much

3. —What‟s your favorite sport?

—Mountain climbing.

—Wow! It‟s exciting, but at the same time you ____________ not forget the danger.

A. can B. may C. must

4. —Would you like to go hiking with us?

—Sorry, I‟m very busy these days.

—____________. Go with us next time.

A. Oh, I‟m sorry, too

B. All right

C. It doesn‟t matter

5. —How is your English?

—Very poor. But I‟m ____________ hard ____________ it now.

A. studying; for B. working; on C. learning; from

6. —The weather by the sea never gets too hot during the summer, ____________?

—Yes, it‟s always cool.

A. doesn‟t it B. isn‟t it C. right

7. —____________ when you first met him?

—It was October 26th, 2004.

A. What day was it B. What time was it C. What was the date

8. —What are you going to the station for?

— My uncle is leaving for New York. And I want to ____________.

A. say goodbye to him

B. meet him

C. say hello to him

9. —Which subject do you like ____________, Chinese or maths?

—Of course maths. And I think it‟s the ____________ interesting of all subjects.

A. best; most B. better; most C. better; more

10. We won‟t go to the Great Wall if it ____________ tomorrow.

A. rain B. rains C. will rain

11. —What‟s ____________ on outside?

—We can see many old people in red and green dancing in the park.

A. going B. happening C. keeping

12. —The meeting starts at eight, ____________ it.

—No, it starts in half an hour.

A. does B. doesn‟t C. isn‟t

13. —Don‟t you think it interesting?

—What?

—The robot can ____________ Mr Mott do everything. And it will tell what it ___________ to his wife, Mrs Mott.

A. watch; sees B. ask; knows C. tell; finds

14. My father ____________ to America on business two days ago.

A. flies B. flew C. will fly

15. —When are you ____________ for your new house?

—Several weeks later, for we don‟t have to move in a hurry.

A. coming B. going C. leaving

二、阅读理解

(A)

Kangaroos live in Australia. They can‟t run and they can‟t walk. But they can jump more than nine metres a time. They can jump very fast—thirty kilometers per hour. Baby kangaroos are very small—only three centimeters long. They can‟t jump and they can‟t see. A young kangaroo is always in its mother‟s pouch for the first six months. They can see when they are nine weeks old. They can jump when they are eight months old.

Camels are perfect for desert. They have two rows of eyelashes to keep sand out of their eyes. They can close their long noses to keep out the sand, too. Camels have humps on their backs—some have one and some have two. Humps do not carry water. They carry food. If a camel is hungry, its hump will get smaller. If a camel is thirsty it will drink a lot of water at one time. A thirsty camel can drink up to one hundred and forty litres of water. If it drinks a lot of water, it won‟t drink again for over a week.

16. Kangaroos ____________ very fast.

A. walk B. run C. jump

17. A young kangaroo leaves its mother‟s pouch when it‟s ____________ old.

A. nine weeks B. eight months C. six months

18. Camels have ____________. So they are not afraid of strong winds in deserts.

A. eyelashes and long noses B. big bodies C. humps

19. There is ____________ in a camel‟s hump.

A. water B. food C. sand

20. A camel drinks ____________.

A. little water

B. water very often

C. a lot of water when it‟s thirsty

(B)

The weather is different in different parts of the world. In some places it is dry, and in others it is wet. If the weather is too dry, the land will not be good for animals or plants. In wet weather there may be too much rain. The rivers may go over their sides. The water may take the bridges away. If there is too much rain, and rivers go over their sides, a lot of people may not have enough food. If there is very dry weather for a long time, the riverbeds may be dry. If they are, there will be neither water nor fish for people. In some other parts of the world the weather may be very cold. There may be snow. If there is a lot of snow, the land and all the trees and buildings will be white. In the winter the days are very short and nights are long. On cold winter nights, when there are no clouds or winds, the sky is very clear. On those nights the moon and stars are very beautiful. People may put on their coats and go for long walks. When they come back to their houses, they may be happy to have hot tea and cake by their stoves(火炉).

21. In different parts of the world, people have ____________ weather.

A. dry B. wet C. quite different

22. When there is too much rain, a lot of people may not have enough food. Why?

A. Because the water in rivers may take the bridges away.

B. Because rivers go over their sides.

C. Because plants can‟t grow well in too much water in the fields.

23. Which of the following is not true?

A. People get little food in too wet or too dry weather.

B. People are afraid of too much water, but not a long time of dry weather.

C. Dry land is not good for plants or animals.

24. People go out for long walks when ____________ during the nights.

A. there are clouds in the sky

B. there are winds

C. there is moonlight

25. In the winter the sun rises ____________ in the summer.

A. later than B. earlier than C. at the same time as

(C)

Many years ago, people wanted to find a way by sea from Europe(欧洲)to China. In 1845, John Franklin left England with 134 men to look for a route(路线).

Franklin‟s ships had everything they needed. They had enough food for three years and thousands of bottles of lemon(柠檬)juice to stop disease. They also had two libraries with 3,000 books and good maps.

Franklin and his men left England on May 19th, 1845 and they sailed(航行)without any problems to Canada. When Franklin arrived at Baffin Bay(海湾) in July 1845, things were going very well. On July 26th, some sailors saw Franklin‟s ships when they were coming to the bay. That was the last time that anyone saw Franklin and his men alive(活着).

In England, many people wanted to know more about Franklin. Some people said that Franklin and his men were still living. Other people said that Franklin was certainly dead.

Jane Franklin was John Franklin‟s wife. She wanted to know what happened to her husband. In July 1857, she sent a group of people to look for her husband‟s ships. A year later, on an island in Baffin Bay, they found one of Franklin‟s sailors—ten years too late. He was lying in a boat near the sea—dead.

Not far away from the dead man, they found a small house. Inside it they found a note. It was the last message from Franklin and his men.

26. What kind of route did Franklin want to find?

A. A route by land from Europe to China.

B. A route by sea from Europe to China.

C. A route by sea from England to Canada.

27. What did they use lemon juice for?

A. Drinking. B. Medicine. C. Food.

28. What happened to Franklin and his men in Baffin Bay?

A. They ate up all the food in their ships.

B. Franklin and some of his men died of disease.

C. An accident happened to their ships.

29. People knew about Franklin‟s death ____________.

A. after 11 years of his death

B. in June, 1847

C. in July, 1857

30. When did Franklin‟s ships get to Baffin Bay?

A. On May 19th, 1845.

B. In July, 1845.

C. In September 1846.

三、完型填空

Mr. Brown and his wife had a small bar near a railway station. The bar didn‟t close ___1___ midnight because people came to drink while they were ___2___ for trains. So the business was good.

At three o‟clock one morning, a man was ___3___ sitting at the table in the bar. He was ___4___. Mr Brown‟s young wife wanted to go to ___5__. She looked ___6___ the bar several times, but the man kept sleeping. Then at last she went to her ___7___ and said to him, “You have tried to wake that man several times, and he isn‟t drinking ___8___. Why haven‟t you sent him away? It‟s too ___9___.”

“Oh, no, I don‟t want to sent him away,” he answered ___10___ a smile. “You know, each time I woke him up, he gave me five pounds. Then he went to sleep again.”

1. ( )

A. when B. until C. as D. while

2. ( )

A. looking B. asking C. waiting D. getting

3. ( )

A. only B. also C. still D. just

4. ( )

A. sleeping B. drinking C. talking D. thinking

5. ( )

A. home B. bed C. school D. work

6. ( )

A. for B. in C. around D. at

7. ( )

A. husband B. table C. room D. bed

8. ( )

A. too B. either C. anything D. everything

9. ( )

A. dark B. early C. cold D. late

10. ( )

A. in B. with C. for D. by

四、词形转换。

1. January is the ______________ month of the year. (one)

2. Please give ______________ an apple. (she)

3. They ______________ the film last night. (not see)

4. My car is ______________ than yours. (big)

5. Li Ming is ______________ boy in his class. (tall)

五、翻译句子。

1. 为什么不骑车去购物呢?

________________________________________________________

2. 好漂亮的公园啊!

________________________________________________________

3. 谢谢你帮我学数学。

________________________________________________________

4. 你最好不要告诉他这件事。

________________________________________________________

5. 我的电脑没出毛病了。

________________________________________________________

六、补全对话。

Mrs Jones: Hello.

Kate: 1. ____________________________ May I speak to Judy?

Mrs Jones: 2. ____________________________ Your call, Judy. It‟s from Kate.

Judy: Hi, Kate.

Kate: Hi, Judy. We‟re going to a dancing party tonight. 3. ____________________________

Judy: I‟d love to. 4. ____________________________

Kate: At the school gate at seven this evening.

Judy: OK. See you then.

Kate: 5. ____________________________

七、阅读与表达,阅读短文,并用完整的句子回答问题。

Hong Kong has about forty public beaches. Some of the beaches are among the best in the world. People can go there for a swim. You can go to most of them by bus. To go to some beaches you must take a boat. There are toilets, changing rooms and places to buy food and drink on most of the beaches.

You will swim there without danger if you remember these instruction:

1. Never swim alone.

2. Never swim after a meal or when you feel hungry or tired.

3. Do not stay in the water too long.

4. Never go out in a boat if you cannot swim.

Remember: A red flag means that it is dangerous for anybody to go into the water. A blue flag means that it is dangerous for children.

1. How many beaches are there in Hong Kong?

______________________________________________________________________

2. Can we reach all the public beaches by bus?

______________________________________________________________________

3. Will you swim after a meal?

______________________________________________________________________

4. What can‟t you do if you can‟t swim?

______________________________________________________________________

5. What does it mean when you see a blue flag?

______________________________________________________________________

八、书面表达

假设你叫李明,下周日是你的生日。你准备在家开一个生日晚会,并邀请部分同学和朋友参加。晚会7:30开始。你家住在牛街50号,乘626路或10路公共汽车在牛街下车。你家就在路的北边。房子是红色的,门是白色的,很容易找到。请根据所给信息,给你的好友林涛写一封不少于50个单词的信。

要点:

1. 邀请他参加晚会;

2. 晚会的时间和地点;

3. 你的地址;

4. 如何到达;

5. 你很希望他来。

注意:信的内容必须包括以上要点,语意连贯,句式规范,信件格式正确。

初二英语寒假作业(九)

一. 单项选择

( )1. We enjoyed at the party yesterday.

A. we B. us C. our D. ourselves

( )2. She can help me my homework.

A. to B. on C. with D. for

( )3. —Coffee is ready. How nice it ! Would you like some?

—Yes, please.

A. feels B. smells C. sounds D. looks

( )4. How about to the radio or reading a newspaper in English?

A. listening B. listen C. to listen D. listened

( )5. There are some people in the park, ?

A. are there B. aren‟t there C. is there D. isn‟t there

( )6. We should always English in class.

A. speak B. to speak C. speaking D. spoke

( )7. They couldn‟t wait their new classmate.

A. meet B. meets C. meeting D. to meet

( )8. Our English teacher to England. She‟ll come back in ten months.

A. have been B. has been C. has gone D. have gone

( )9. It‟s a good idea your vocabulary notebook every day.

A. check B. to check C. checking D. checked

( )10. The teacher a gift, but he didn‟t it.

A. accepted; accept B. accepted; receive

C. received; receive D. received; accept

( )11. —I called you yesterday evening, but there was no answer.

—Oh, I am sorry, I dinner in my friend‟s home.

A. have B. had C. was having D. have had

( )12. —What are you going to do in Spring Festival?

—I am not sure. I make a trip to Tianjin with my friends.

A. will B. possible C. might D. must

( )13. —Could you give me a hand, please?

—Sure. What would you like me ?

A. do B. to do C. doing D. does

( )14. The film has for about twenty minutes.

A. started B. begun C. been D. been on

( )15. — ?

—It‟s snowy.

A. What‟s the weather like B. What does she look like

C. What‟s the matter D. How is it about

二. 完型填空

Everyone in our school loves sports. Every morning we get up, we do morning exercises. After the second class we do exercises again. We only have classes twice a week, but we do physical training (锻炼) at five every afternoon. The most popular sport is basketball. The enjoy playing it and many of the girls like it, too. popular sport is football and in every class there a lot of football fans (球迷). volleyball is played when the weather is fine. We have school teams in

basketball, football and volleyball. Our teams often friendship (友谊) matches with the teams from other schools. When there is a match, of us go to watch it and cheer (喝彩) our side on.

Beside (除„„之外) ball games, some of us like track-and-field events (田径项目), we often practice running, jumping and throwing. Every term we have tests in these events and once a year we hold a sports meeting.

Sports help us to keep .

( )1. A. while B. when C. before D. after

( )2. A. English B. Chinese C. P.E. D. music

( )3. A. teacher B. boys C. students D. people

( )4. A. Some other B. Other C. Another D. The other

( )5. A. are B. is C. were D. was

( )6. A. Never B. Often C. Usually D. Always

( )7. A. see B. have C. look at D. watch

( )8. A. several B. few C. a little D. many

( )9. A. because B. but C. or D. and

( )10. A. thirsty and hungry B. healthy and happy

C. fat and tired D. thin and weak

三. 阅读理解

A

We each have a memory. That‟s why we can still remember things after a long time. Some people have very good memories and they can easily learn many things by heart, but some people can only remember things when they say or do again and again. Many of the great men of the world have got surprising memories.

A good memory is a great help in learning languages. Everybody learns his mother language when he is a small child. He hears the sounds, remembers them and then he learns to speak. Some children are living with their parents in foreign countries. They can learn two languages as easily as one because they hear, remember and speak two languages every day. In school it is not so easy to learn a foreign language because the pupils have so little time for it, and they are busy with other subjects, too.

But your memory will become better and better when you do more and more exercises.

( )1. Some people can easily learn many things by heart because .

A. they always sleep well B. they often eat good food

C. then read a lot of books D. they have very good memories

( )2. Everybody learns his mother language .

A. at the age of six B. when he is a small child

C. after he goes to school D. when he can read and write

( )3. Before a child can speak, he must .

A. read and write B. make sentences

C. hear and remember the sounds D. think hard

( )4. In school the pupils can‟t learn a foreign language easily because .

A. they have no good memories B. they have no good teachers

C. they don‟t like it D. they are busy with other subjects

( )5. Your memory will become better and better .

A. if you have a lot of good food

B. if you do more and more exercises

C. if you do morning exercises every day

D. if you get up early

B

The following are three ads (广告):

Driver Wanted

1. Clean driving license

2. Good-looking

3. Age over 25

Apply (申请)to: Capes Taxi, Shenzhen Tel: 0755-6561382

Air Hostess (空姐) Wanted

1. Age between 20 and 33

2. Height from 1.6m to 1.75m

3. Two foreign languages

4. College graduate

Apply to: China Airlines, Beijing

Tel: 010-88488970 Teacher Needed For private language school Teaching experience necessary Apply to: Instant Languages Ltd, Dalian Tel: 0411-4313861

( )6. If you want to work in the south, you can apply for a job as .

A. a driver B. an air hostess C. a teacher D. all of the above

( )7. You may call when you wish to be a teacher.

A. 0755-6561382 B. 0411-4313861

C. 010-88488970 D. A and B

( )8. Mary, aged 26, knows English and Japanese, which job can be given to her?

A. Driving for China taxi. B. Working for China Airlines.

C. Teaching at instant languages Ltd D. None of the above.

( )9. What prevents Jack, an experienced taxi driver, working for Capes Taxi?

A. Liking beer and wine.

B. Breaking traffic rules.

C. Being unable to speak a foreign language.

D. Not having college education.

( )10. Which of the following is not mentioned in the three ads?

A. Height B. Age C. Language D. Health

C

Happiness is for everyone. You don‟t need to care about those people who have beautiful houses with large gardens and swimming pools or those who have nice cars and a lot of money and so on. Why? Because those who have big houses may often feel lonely and those who have cars may want to walk on the country roads at their free time.

In fact, happiness is always around you if you put your heart into it. When you are in trouble at school, your friends will help you; when you study hard at your lessons, your parents are always taking good care of your life and your health; when you get success, your friends will say congratulations to you; when you do something wrong, people around you will help you to correct it. And when you do something good to others, you will feel happy, too. All these are your happiness. If you notice a bit of them, you can see that happiness is always around you.

Happiness is not the same as money. It is a feeling of your heart. When you are poor, you can also say you are very happy, because you have something else that can‟t be bought with money. When you meet with difficulties, you can say loudly you are very happy, because you have more chances to challenge (向„„挑战) yourself. So you can not always say you are poor and you have bad luck. As the saying goes, life is like a revolving (旋转) door. When it closes, it also opens. If you take every chance you get, you can be a happy and lucky person.

( )11. Those who have big houses may often feel .

A. happy B. lonely C. free D. excited

( )12. When you fall down in a PE class, both your teacher and your classmates will .

A. laugh at you B. play jokes on you

C. quarrel with you D. help you up

( )13. What will your friends say to you when you make great progress?

A. Oh, so do I. B. Congratulations.

C. Good luck. D. It‟s just so-so.

( )14. Which idea is NOT right according to the passage?

A. People who have cars would never like to walk in the open air.

B. You can get help from others when you make mistakes.

C. You can still be a happy person even if you have little money.

D. Happiness is always around you though difficulties come towards you.

( )15. Which of the following is this passage about?

A. Bad luck. B. Good luck. C. Happiness. D. Life.

. 四、从括号中选择恰当的词填空

1. I‟m very thirsty. Please 2. Don‟t go (through, across) the road when the red lights are on.

3. I was very 4. These 5. My father made me (go, to go) to bed early.

五. 完成句子

1. 当你到这个国家学一门外语就很容易。

2. 学习英语用了我很长时间。

3. 你能给我看看那份地图吗?

Could you ?

4. 这件毛衣摸上去很柔软。

This sweater .

5. 你想要听乡村音乐吗?

六. 看图完成对话

W: Excuse me, where is the nearest supermarket, please?

M1: Sorry, I don‟t know. W: Thank you all the same. Excuse me, ?

M2: Go down this street. . Then you‟ll see the super market. It‟s between And you can go there by bus.

W: ?

M2: Over there on the other side of the street.

W: I see. Thank you very much.

七. 阅读与表达

I was on my way to the Taiyetos Mountains. The sun was going down when my car broke down near a faraway village. I was thinking about my bad luck and didn‟t know where I was going to spend the night when the villagers came up to me, asking me to go to the houses of their own as if I were their best guest (客人). At last I went into an old woman‟s home. She lived alone in a little house. While I was having a rest, the villagers took my car with their cows to a nearby car repair shop.

I had noticed (注意到) there were three chickens walking in the garden that evening but only two were left alive (活着的) later. One of them was on my dinner table in the evening. Other villagers brought me a lot of food to eat. Though I didn‟t know their language, we drank happily together till far into the night. The following day the villagers brought back my car.

When the time came for me to say goodbye to my friends in the village, I wanted to give some money to the kind old woman, as I gave her so much trouble. She got angry and said she couldn‟t take the money.

根据短文内容,回答下列问题。

1. Where was the writer traveling?

2. When was his car broken?

3. Where had one of the old woman‟s chickens gone?

4. Why couldn‟t they talk to each other?

5. Why didn‟t the old woman take the money?

八. 书面表达

校刊要求每位同学写一篇短文介绍自己最要好的朋友,请以“My best friend, Mike”为题写好介绍,词数不少于60个。短文应包括下列所有要点提示:

1. 他的外貌特征:高而瘦,圆脸,大眼睛,小鼻子,脸上总带着笑容„„ 2. 他的个性特点:诚实,保守秘密,乐于助人,而且幽默„„ 3. 他的理想:长大想当一名运动员„„

初二英语寒假作业(十)

一. 选择填空

1. I‟d like to go shopping with you, I‟m too busy today. A. and B. but C. or D. so 2. Welcome to our hotel. It‟s one in the city. A. good C. better C. best D. the best 3. The children will climb the hill if it tomorrow. A. won‟t rain B. didn‟t rain C. doesn‟t rain D. isn‟t raining 4. They can go to school A. under B. with C. in D. to 5. —What good weather! Let‟s go boating on the lake. — A. That‟s all right B. Thank you very much C. It doesn‟t matter D. That‟s a good idea 6. —What‟s your brother doing in his room now? —He A. makes B. made C. is making D. will make 7. A. How many B. How often C. How much D. How far

8. It five years since I saw him last time. A. was B. are C. is D. were 9. I‟m late, but I on time tomorrow. A. came B. come C. will come D. have come 10. She has got a cold but she will get well A. after B. before C. in D. for 11. —How about your trip to the seaside? —Great! I enjoyed A. swimming B. to swim C. swim D. swims 12. —We can go to Beijing Railway Station by bus. —Why not A. walk B. walking C. to walk D. walked 13. —He didn‟t come to the meeting, he? — A. didn‟t; Yes B. did, Yes C. didn‟t; No D. did; No 14. The student Jane is from England. A. naming B. to name C. names D. named 15. —I haven‟t seen you for quite a long time. —I am busy A. write B. writing C. written D. to write 16. The students are very tired. Let them a rest. A. to have B. have C. having D. had 17. Mr. Yu often has sports on Sunday morning, A. does B. doesn‟t C. has D. hasn‟t 18. —you —Yes. I A. Have, returned; returned B. Do, return; returned C. Will, return; have returned D. Did, return; returned 19. My family will go to Beidaihe th. A. in B. at C. on D. by 20. We a meeting at eight yesterday. A. are having B. will have C. were having D. had

21. After having the soup I cooked, mum said, “Wow, it A. tastes B. seems C. smells D. feels 22. —You have caught a cold. Is it very serious? —No, A. nothing serious B. serious nothing C. anything serious D. serious anything 23. The sweater feels very . A. soft and comfortable B. softly and comfortably C. softly and comfortable D. soft and comfortably 24. The old man told the boys football on the street. A. not play B. not to play C. don‟t play D. didn‟t play 25. What will the weather A. like B. be likely C. be D. be like

26. I took Mr. Black to the hospital this morning, A. because B. and C. why D. so 27. —are you going in Beijing?

—We are going to the Summer Palace and the Great Wall. A. Where B. What C. Why D. When 28. Could you help A. I, my B. me, me C. me, my D. my, I 29. The good news made us very . A. excited B. exciting C. to excite D. be excited 30. —Must I finish that work today?

—No, you A. mustn‟t B. needn‟t C. shouldn‟t D. couldn‟t

二. 完型填空。

A

Mary was quite busy yesterday. She up before 7:00 in the morning. She her face very quickly and had some milk and bread for . It was a fine day. She went to early. She had four classes in the morning, maths, Chinese, English and PE. She didn‟t go back home for lunch. She just had it at school. She had an easy lunch. Then she had a little rest, but she worked very hard in class . School was over at half past four. She played basketball after school with her classmates and then went home. home she bought a new golden pen. When she home she had a short rest. Because her mother was not at home, she had to cook supper quickly and supper. Then she did all her homework and went to before ten. 1. A. get .B. gets C. got D. getting 2. A. washed B. cleaned C. washes D. cleans 3. A. lunch B. supper C. breakfast D. dinner 4. A. cinema B. office C. school D. bank 5. A. all day B. the all day C. all the day D. all a day 6. A. To her way B. By her way C. On her way D. On way 7. A. got B. reached to C. reached D. gets 8. A. did some cleaning B. doing some cleaning C. clean a house D. to clean the house 9. A. watched B. read C. saw D. looked 10. A. bed B. beds C. the bed D. a bed

B

Cars are very popular in America. When the kids are fourteen years old, they dream of having their own work after school to a car. In most places people learn to drive in high school. They have to take a test to get a license. Learning to drive and getting a driver‟s may be one of the most exciting things in their lives. For many, that piece of paper is an important symbol that they are now grown-ups.

Americans seem to love their cars almost more than anything else. People almost never go to see a doctor when they are But they will take cars to a “hospital” at the smallest sign of a problem. At weekends, people most of the time in washing and waxing (打蜡) their cars. For some families it is not enough to have car. They often have two or even three. Husbands need a car to go to work. Housewives need a car to go shopping or to take the children to school or 1. A. cars B. computers C. bikes D. houses 2. A. borrow B. buy C. lend D. sell 3. A. old B. tall C. strong D. young 4. A. language B. listening C. driving D. body 5. A. address B. book C. name D. license 6. A. sick B. healthy C. pleased D. angry 7. A. his B. her C. your D. their 8. A. cost B. take C. spend D. pay 9. A. no B. one C. some D. several 10. A. other B. another C. others D. else

三. 阅读理解

A

It was 10:30 at night. Mrs. White put down her book and went into her 14-year-old son‟s room. Tom was sitting in front of a bright computer screen on which a colourful dinosaur jumped and shouted.

“Oh, Tom! You‟re still playing. You must stop and go to bed now. If not, you‟ll be very tired tomorrow.” Said Mrs. White. “But I‟ve nearly beaten the dinosaur.” Said Tom.

“You‟re always playing on that computer. You spend more time with this machine than with us,” she said with a smile. “What‟s so special? Show me what it can do!”

Tom was very excited. “I think it‟s a great computer, Mum!” he said happily, “The hardware is good. There‟s so much memory that I can put in a lot of software programs. This game, DINOSAURSPLAYER, is my favorite. It‟s about a super boy exploring the moon with the help of his knowledge in maths and physics. It‟s interesting and helpful. Also, I can download lots of information from the Internet so that I can know what is happening at home and abroad. Now let me show you!” Tom began pressing keys on the keyboard. The screen changed in answer to his orders.

“Oh, Tom,” laughed Mrs. White. “I‟m sure it‟s a wonderful computer, but I‟m afraid I don‟t know much of what you‟re talking about.” 1. When Mrs. White entered Tom‟s room, he was

A. lying in bed

B. standing in front of the computer C. playing computer games D. reading a book

2. From the passage we know that the story happened A. before Tom went to school B. before Tom went to bed C. just after Tom had supper D. after Tom slept for hours

3. Tom was very excited because A. he beat the dinosaur

B. he was worried about his computer game

C. his mother began to feel interested in the computer D. his mother asked him to go to bed 4. What will be the possible end of the story? A. Tom will never play computer games again.

B. Mrs. White will never ask Tom to go to bed early. C. Tom will not get on well with his mother.

D. Mrs. White and Tom will understand each other better.

B

Over 1,400 university students from 37 countries and regions came to China for the 2006‟s WorldMUN meeting (世界大学生模拟联合国大会). The meeting was held in Peking University from March 27th to 31st, 2006.

The idea of the WorldMUN began in 1991. A group of students at Harvard University wanted to make a new meeting to bring together the students who were interested in the world. They wanted to discuss different problems around the world. The WorldMUN is based on Last year, it was held in Scotland.

This was the first time the WorldMUN meeting was held in Asia. It was also the biggest one in history. The students came from the best universities around the world. During the five-day meeting, they did everything that people in the UN do. Though their skin colors and cultures were different, they had the same wish to make the world better. 5. The 2006‟s WorldMUN meeting was held in A. Scotland B. Harvard University C. Poland D. Peking University

6. The word “compromise” in the passage means in Chinese. A. 友谊 B. 理解 C. 和解 D. 屈服 7. Which of the following is NOT true?

A. The students want to travel around the world. B. The students are interested in the world.

C. The students want to discuss different problems. D. The students wish to make the world better. 8. The best title of the passage is . A. The First WorldMUN Meeting B. The WorldMUN Meeting C. University Students

D. Problems Around the World

C

The modern summer Olympic Games which was held in Barcelona, Spain in 1992, was the 25th since it was first held in Athens, Greece in 1896. Do you know the dates and the places of all the 25 Games, and also the total of gold medals (金牌总数) the first three

*Note: GDR is East Germany.

9. By the year 1992, the modern summer Olympic Games had been held times in USA. A. one B. two C. three D. four

10. How many gold medals did East Germany win in the summer Olympic Games? A. 124. B. 144. C. 198. D. 200. 11. From the data (资料) we can see that .

A. every country should hold the Olympic Games in turn

B. any Olympic Games should not be called off (取消) for any reason C. no peace, no sports

D. if there is a war anywhere, the Olympic Games must be stopped

D

Maybe you know floods, droughts, earthquakes, sandstorms and so on. But have you ever heard of typhoons?

Typhoons are some of the worst storms, usually around the Pacific Ocean and the South China Sea, most happen in July, August and September.

How does a typhoon happen? When lots of sea water gets hot in the summer sun, it evaporates into the air. This makes the air hotter. After the air gets warmer, it starts to move quickly, making wind. The wind goes in circles, and it keeps moving higher in the sky. The warmer the air gets, the quicker the wind moves. And when the wind moves faster than 30 meters a second, a typhoon begins.

A typhoon has two parts. One is called the “eye”. In the eye, the wind does not move so fast. The other part is the wall of clouds around the eye. This is where the strongest and heaviest rains are.

Typhoons are very dangerous. In 2004, Typhoon Yunna killed 164 people in Zhejiang, and 24 people were missing. In May 2006, Typhoon Pearl hit Guangdong and Fujian, and thousands of houses were damaged.

What to do when a typhoon hits?

Stay inside, Close all the windows and stay away from them.

Try to bring all of your things inside. Strong winds could even blow away your bikes! Listen to the radio or TV for important information. If you‟re told to go to a safer place, do so right away. 12. Typhoons often happen . A. in the hot season B. in the cold season C. only in China D. under the sea

13. Which picture can tell meaning of the underlined sentences “…, it starts to move higher up into the sky. Then, cooler air around it rushes in.”?

14. When a typhoon comes, we cannot A. close all the windows

B. listen to typhoon news

C. stay outside with our bikes

D. stay inside the strong house

15. Which of the following is TRUE according to the passage?

A. The warmer the air gets, the slower the wind moves.

B. This year Typhoon Peal killed 164 people in Zhejiang.

C. The strongest winds and heaviest rains are in the eye of typhoon.

D. A typhoon will happen when the wind goes faster than 30 meters a second.

四. 根据句意,用所给单词的适当形式填空。

1. Mr. Smith gave us some (建议) on how to learn English.

2. Liu Shanzi can‟t get an 教育) because his family is poor.

3. Pandas live in the forests and mountains of (西南) China.

4. It is 可能) that we can see the film this weekend.

5. We learn a lot at the 博物馆).

6. Pandas mainly live on . (bamboo)

7. The fishermen are told to be more careful on days. (wind)

8. Of all the subjects, which do you think is (difficult) to learn?

9. Yan Liwei returned to the earth (safe).

10. Please make a (shop) list before you go out to buy things.

五. 动词的适当形式填空

1. I don‟t need 2. —When —The day after tomorrow.

3. —Who (knock) at the door?

—It‟s me.

4. He (use) his bike for twenty years, but he doesn‟t want to buy a new one.

5. —When the plane (leave), sir?

—I‟m not sure. Maybe five minutes ago.

6. In many parts of China, it often (rain) a lot in summer.

7. When I visited them last night, they 8. The teacher was busy 9. —Where is Mr. White? I haven‟t seen him for a few days.

—He 10. Would you please

六. 句型转换 (对画线部分提问)

is the Yellow River?

2. The old man knew that too much salt was harmful (改为否定句)

The old man that too much salt was harmful.

3. This dress cost her 90 yuan. (改为一般疑问句)

4. Tom has cleaned the room, ? (反意疑问句) (划线部分提问)

the little boys play football yesterday?

七. 翻译句子

1. 无论刮风下雨,Tom总是第一个到校。

Whether it is windy or rainy, Tom is always the first to school.

2. 班会已经开了半个小时。

The class meeting has for half an hour.

3. 春天树叶变绿了。

The leaves in spring.

4. 我能把你做的空间站模型拿给我父母看吗?

your space station to my parents?

5. 夏威夷因美丽的海滩而闻名。

Hawaii is its beautiful beaches.

八. 根据语境,用适当的语言完成对话。

A: It‟s a fine day today. Let‟s go boating.

B: Sorry, .

A: All right. But which shop are you going to?

B: Mr. Wang‟s shop.

A: But it‟s very far and we have only one bike.

B: Let‟s put it away and A: Oh, it‟s half past two. Let‟s hurry. B: Why?

A: There is going to be a football match at half past four. .

B: All right. Let‟s go at once.

九. 阅读回答问题。

28 High Road

Oxford

Sept. 20, 2006

Dear John,

Thank you very much for your letter. I‟m glad you enjoyed your holiday with me.

We enjoyed having you and your sister here. We hope that you will both come again next year. Perhaps you can stay longer next time you come. A week is not really long enough, is it?

I hope you are both working hard now that you‟re in Grade Three. I shall have to work hard next year when I am in Grade Three. Tom and Ann will be in Grade Three the year after next.

They went on a picnic yesterday but I didn‟t go with them because I hurt my foot and I couldn‟t walk very well. They went to an island and enjoyed themselves there. Do you still remember the island? That‟s where all five of us spent our holiday.

Tom, Ann and I send our best wishes to Betty and you. We hope to see you soon.

Yours sincerely,

Peter

根据短文回答问题,用整句。

1. How long did their holiday last this year?

2. Who stayed with Peter for the holiday?

3. When did Peter write the letter?

4. Who is in Grade Two now?

5. Why didn‟t Peter go to the island for picnic?

十. 写作

请你参考所给信息,写一篇关于你的同学或朋友的小短文。

要求:

1. 短文中不得出现你的真实姓名。

2. 必用所给信息,可适当发挥。

3. 词数:60~80词。

信息:Zhang Li: 14; tall/ short; thin/ strong; kind; favorite sport; listen to music; read books…


相关内容

  • 初二寒假英语作业
  • 初二英语寒假作业指导 一. 完成<寒假生活指导>英语部分所有内容.时间:______ 家长签字:_____ 二. 搜集5条英语谚语或格言并记住,查出单词的读音和意思,并背诵. 时间:______________ 评价:___________________________________ ...

  • 初二寒假学习计划
  • 充实,有意义的寒假,特制定此寒假学习计划 1.每天足量学习5小时,内容包括:寒假各科作业:语文阅读、数学练习、英语听力等 2.每日早晨 英语听力或口语30分钟 3.中午保证一小时休息,下午学习或外出体育活动(运动能使人头脑更清醒) 4.做数学竞赛书《初中数学竞赛新课标同步辅导(初二)》(这本书真的很 ...

  • 初二级优秀寒假作业名单
  • 初二级优秀寒假作业名单 语文 初二(1)班 谭晓晴 龚晓婷 黄妙瑜 卢瑞意 梁伟岚 廖梓锨 王茵霖 廖佩珊 廖秀莹 叶楚枫 蔡玮然 吴颖琳 梁趣诗 杨颖锭 张惠淇 初二(2)班 缪文婷 曹海镟 萧咏晖 郭绮琪 林金霞 邓雅文 梁淑欣 尹翠仪 林秀侨 卢倩仪 霍秀烽 潘晓晴 张敏炜 萧耀澄 廖颖镱 初 ...

  • 初三新生应该提早制订中考总目标
  • 初三新生应该提早制订中考总目标 更新时间:2009-8-20 9:39 [大?中?小]????还有十来天就开学了,新初三生即将开始备战中考,有关心理专家提醒,除了知识.身体上的准备,心理上的准备也非常重要,学生要制订合理目标并有计划地实施.????合理定位锁定目标????不少新初三生已经开始准备迎接 ...

  • 家教寒假实践报告
  • 窗外此起彼落的鞭炮声震耳欲聋,到处都洋溢着过年的喜庆祥和气氛. 转眼间,已经是元宵节了,回想去年年末为期末考奋战的辛劳及对放假回家过年的期待,不禁感慨时间流逝之飞快. 很快,我们又要结束寒假,回到学校继续学习了.假期的末尾,是时候对这个寒假做个总结了. 我们毕竟是四年级的学生了,还有一年半,我们就要 ...

  • 2014年寒假做家教实践报告
  • 窗外此起彼落的鞭炮声震耳欲聋,到处都洋溢着过年的喜庆祥和气氛. 转眼间,已经是元宵节了,回想去年年末为期末考奋战的辛劳及对放假回家过年的期待,不禁感慨时间流逝之飞快. 很快,我们又要结束寒假,回到学校继续学习了.假期的末尾,是时候对这个寒假做个总结了. 我们毕竟是四年级的学生了,还有一年半,我们就要 ...

  • 四年级的新学期打算
  • <四年级的新学期打算>400字 结束了一个愉快又漫长的暑假,新的学期即将又开始了,转眼间我就快是一位四年级的小学生了.课程跟三年级一样,但作业量也会大大增加了许多,我们的老师也跟三年级一样,语文老师是聂老师,数学老师是李老师,体育老师是倪老师. 这个学期我准备把自己马虎.粗心大意的毛病给 ...

  • 2014寒假作业(完整版)
  • 2014年初一年级寒假作业 [语文] 用书写来培养习惯 一. 小字练习. 要求:准备一本小字本:根据个人手中字帖,认真临写一面,完成后标注当天日期,总共完成12面. 用精读来感悟生命 二. 读书笔记. 阅读<朝读美文>并认真完成读书笔记.读书笔记内容如下: A.为文中的生字词注音: B. ...

  • 2013秋期末工作安排
  • 2013-2014学年度上学期期末教学工作安排 各教研组.各年级组: 本学期即将结束,为了有条不紊地做好本学期后期阶段的各项工作,现将期末工作安排如下,望遵照执行. 一.新课结束.期考备考要求: 1.各学科结束新课时间为18周(1月1日前):18周周四至19周进行复习(每科至少应该有一周课的时间进行 ...

  • 2011初二八年级上寒假作业答案深圳
  • 本文由云鹤飞天之梦贡献 doc文档可能在WAP端浏览体验不佳.建议您优先选择TXT,或下载源文件到本机查看. 2011 初二八年级上寒假作业答案 深圳(罗湖区) 深圳(罗湖区) 语文~ 第一单元 1.给下列加点字注音 阻遏(ě) 仄(zè)歪转弯抹(mò)角拂(fú)晓赃(zāng)物悠(yōu)闲 ...